Adv Patho Study guide 2

Réussis tes devoirs et examens dès maintenant avec Quizwiz!

Release of which of the following humoral factors will result in vasodilation? A) Norepinephrine B) Angiotensin II C) Serotonin D) Histamine

Ans: D) Histamine

A client with a long history of "hay fever" has recently begun a series of immunotherapy (allergy shots). This treatment potentially will achieve a therapeutic effect by: A) Blocking cytokine release from sensitized mast cells B) Preventing mast cells from becoming sensitized C) Causing T cells to be sequestered in the thymus for longer periods D) Stimulating production of IgG to combine with antigens

Ans: D) Stimulating production of IgG to combine with antigens feedback In immunotherapy, injected antigens stimulate production of high levels of IgG, which acts as a blocking antibody by combining with the antigen before it can combine with the cell- bound IgE antibodies. This therapy does not block cytokine release, prevent mast cell sensitization, or change the maturation process of T cells.

Although both are characterized by ischemia, Raynaud phenomenon is caused by _________, and thromboangiitis obliterans is caused by: A) occlusion; compression. B) thrombi; vasoconstriction. C) vasculitides; hypertension. D) vasospasm; inflammation.

Ans: D) vasospasm; inflammation.

A 5-year-old child is experiencing itchy, watery eyes and an increased respiratory rate with some inspiratory wheezes. He has been outside playing in the yard and trees. The mother asks, "Why does he get like this?" The health care worker's best response is: A) "This is what we call a type I hypersensitivity reaction and usually occurs a few minutes after exposure to his allergen. It is primarily caused by mast cells in his body." B). "Because his allergy is related to something in his environment, the best thing you can do is try to keep him indoors as much as possible." C) "This sounds like he is on his way to having an anaphylactic reaction and you need to get a prescription for an EpiPen to decrease his response to monocytes." D) "This is pretty common in children. He is just getting used to all the allergens in the air. I suggest you just give him a shower after every time he plays outside."

Ans; A Feedback: The immediate response to allergen exposure is mast cell degranulation and release of mediators such as histamine and acetylcholine. Monocytes respond as part of the acute immune response. There is no truth to this being an anaphylactic reaction. Showers may help, but the underlying cause is the mast cell degranulation and the release of preformed mediators.

A patient who lives with a diagnosis of angina pectoris has taken a sublingual dose of nitroglycerin to treat the chest pain he experienced while mowing his lawn. This drug has resulted in a release of nitric oxide, which will have what effect? A) Smooth muscle relaxation of vessels B) Decreased heart rate and increased stroke volume C) Increased preload D) Reduction of cardiac refractory periods

Ans; A) Smooth muscle relaxation of vessels

A patient with a diagnosis of secondary hypertension has begun to experience signs and symptoms that are ultimately suggestive of decreased cardiac output. Which of the following factors that determine cardiac output is hypertension likely to affect most directly? A) Preload B) Afterload C) Contractility D) Heart rate

Ans; B) Afterload

The course of any infectious disease progresses through several distinct stages after the pathogen enters the host. Although the duration may vary, the hallmark of the prodromal stage is: A) tissue inflammation and damage. B) initial appearance of symptoms. C) progressive pathogen elimination. D) containment of infectious pathogens.

Ans; B) initial appearance of symptoms.

Heart muscle differs from skeletal muscle tissue by being able to generate: A) contractions. B) calcium influx. C) action potentials. D) sarcomere binding.

Ans; C) action potentials.

Whatever the mechanism of entry, the human- to-human transmission of infectious agents is directly related to the: A) source of contact. B) site of infection. C) number of pathogens absorbed. D) virulence factors.

Ans; C) number of pathogens absorbed.

Although bacterial toxins vary in their activity and effects on host cells, a small amount of gram-negative bacteria endotoxin: A) is released during cell growth. B) inactivates key cellular functions. C) uses protein to activate enzymes. D) in the cell wall activates inflammation.

Ans; D) in the cell wall activates inflammation.

An 86-year-old male client is disappointed to learn that he has class II heart failure despite a lifelong commitment to exercise and healthy eating. Which of the following age-related changes predisposes older adults to developing heart failure? A) Increased vascular stiffness B) Orthostatic hypotension C) Increased cardiac contractility D) Loss of action potential

A feedback Increased vascular stiffness in older adults causes a progressive increase in systolic blood pressure with advancing age, which in turn contributes to the development of left ventricular hypertrophy and altered diastolic filling. A loss of action potential does not typically accompany aging, and contractility tends to decrease as a result of cardiac stiffness. Orthostatic hypotension is neither a normal age-related change nor a cause of heart failure.

When explaining the body's compensatory mechanisms to maintain a normal pH, the health care provider knows that the renal system: A) Works slower than the respiratory system, going into action 1 to 2 days after H+ remain elevated B) Will absorb more bile acids to try to normalize elevated H+ levels C) Will absorb more Na+ and water to dilute the elevated H+ in an effort to normalize pH D) Waits until the lungs have increased the respiratory rate to try to blow off excess CO2

A feedback Only the kidney can eliminate hydrogen from the body. Virtually all the excess H+ excreted in the urine are secreted into the tubular fluid by means of tubular secretory mechanisms. The ability of the kidneys to excrete large amounts of H+ in the urine is accomplished by combining the excess ions with buffers in the urine. The three major urine buffers are HCO3-, phosphate (HPO42-), and ammonia (NH3). An important aspect of this buffer system is that the deamination process increases whenever the body's hydrogen ion concentration remains elevated for 1 to 2 days.

Assessment of an elderly female client reveals the presence of bilateral pitting edema of the client's feet and ankles and pedal pulses that are difficult to palpate. Auscultation of the client's lungs reveals clear air entry to bases, and the client's oxygen saturation level is 93%, and vital signs are within reference ranges. What is this client's most likely health problem? A) Right-sided heart failure B) Pericarditis C) Cardiogenic shock D) Cor pulmonale

A Feedback: A major effect of right-sided heart failure is the development of peripheral edema. A client who is in shock would not have stable vital signs. Cor pulmonale would be accompanied by manifestations of lung disease. Pericarditis is an inflammation of the pericardium exhibited by fever, precordial pain, dyspnea, and palpitations.

A client who has been diagnosed with Addison disease will likely experience which of the following lab results related to the absence of aldosterone? A) Serum potassium levels elevated B) Increased serum sodium levels C) Elevated creatinine levels D) Decreased serum chloride levels

A Feedback: Aldosterone exerts a strong influence on potassium secretion in the distal and collecting tubules. In the absence of aldosterone, as occurs in Addison disease, potassium secretion is markedly decreased, causing blood levels to increase. In the presence of aldosterone, almost all the sodium in the distal tubular fluid is reabsorbed, and the urine essentially becomes sodium free. In the absence of aldosterone, virtually no sodium is reabsorbed from the distal tubule, and excessive amounts of sodium are lost in the urine.

A client who has developed stage 3 renal failure has been diagnosed with high phosphate levels. To avoid the development of osteodystrophy, the physician may prescribe a phosphate-binding agent that does not contain: A) Aluminum B) Calcium carbonate C) Calcium acetate D) Sevelamer hydrochloride

A Feedback: Aluminum-containing antacids can contribute to the development of osteodystrophy, whereas calcium-containing phosphate binders can lead to hypercalcemia, thus worsening soft tissue calcification, especially in persons receiving vitamin D therapy. Sevelamer hydrochloride is a newer phosphate-binding agent that does not contain calcium or aluminum.

Bronchial circulation differs from the pulmonary circulation by providing blood for the: A) Conducting airways B) Alveolar gas exchange C) Mediastinal and pleural space D) Intrapulmonary pressure balance

A Feedback: Bronchial circulation in the lungs provides blood to the airways. Pulmonary circulation provides the blood for gas exchange and to lung tissue other than the airway tissue. Intrapulmonary pressure is the gas pressure in the airways, which is unrelated to blood circulation.

An 86-year-old female client has been admitted to the hospital for the treatment of dehydration and hyponatremia after she curtailed her fluid intake to minimize urinary incontinence. The client's admitting laboratory results are suggestive of prerenal failure. The nurse should be assessing this client for which of the following early signs of prerenal injury? A) Sharp decrease in urine output B) Excessive voiding of clear urine C) Acute hypertensive crisis D) Intermittent periods of confusion

A Feedback: Dehydration and its consequent hypovolemia can result in acute renal failure that is prerenal in etiology. The kidney normally responds to a decrease in GFR with a decrease in urine output. Thus, an early sign of prerenal injury is a sharp decrease in urine output. Postrenal failure is obstructive in etiology, and intrinsic (or intrarenal) renal failure is reflective of deficits in the function of the kidneys themselves.

As chronic kidney disease progresses, the second stage (renal insufficiency) is identified by: A)Decrease in GFR of 60 to 89 mL/minute/1.73 m2 B) Decrease in GFR to 30 to 59 mL/minute/1.73 m2 C) GFR decrease to 15 to 29 mL/minute/1.73 m2 D) Diminished GFR to less than 15 mL/minute/ 1.73 m2

A Feedback: Diminished renal reserve is characteristic of renal insufficiency, when labs remain normal but there is renal insufficiency. Only the second stage, formerly known as renal insufficiency, is characterized by a decrease in GFR of 60 to 89 mL/minute/1.73 m2. The other choices represent stage 3, 4, and 5, respectfully.

Although urinary obstruction and urinary incontinence have almost opposite effects on urination, they can both result from: A) Bladder structure changes B) Bladder wall atrophy C) Micturition reflex spasms D) Bladder distensibility loss

A Feedback: Disorders of lower urinary tract structure and function include urinary obstruction with retention or stasis of urine and urinary incontinence with involuntary loss of urine. Both types of disorders can have their origin in the structures of the lower urinary tract or in the neural mechanisms that control their function. Urinary incontinence can result from loss of bladder distensibility. Chronic outlet obstruction can cause bladder wall hypertrophy. Incontinence can result from reflex spasms, leading to segmental reflex bladder control instead of micturition center control.

While taking a history from an adult client newly diagnosed with renal cell cancer, the nurse can associate which of the following high-risk factors with the development of this cancer? A) Heavy smoking B) Inherited renal disease C) Adrenal medulla tumors D) Anorexia/bulimia disorder

A Feedback: Epidemiologic evidence suggests a correlation between heavy smoking and kidney cancer. Obesity also is a risk factor; particularly in women. The risk of renal cell carcinoma also is increased in persons with acquired cystic kidney disease associated with chronic renal insufficiency. Although the adrenal gland is adjacent to the kidney, primary adrenal tumors are unrelated to renal cell disease.

A client fell off a ladder and sustained a spinal cord injury that has resulted in bladder dysfunction. During the period immediately after the spinal injury, spinal shock develops and the bladder displays what type of function? A) Atonic B) Spasmodic C) Uninhibited D) Hyperactive

A Feedback: Immediately following spinal cord injury, a state of spinal shock develops in which all reflexes, including the micturition reflex, are depressed. During this stage, the bladder becomes atonic and cannot contract. Spasmodic, hyperactive, and uninhibited bladder function occurs after the acute stage of spinal cord injury and during the recovery stage.

In the ICU setting, clients who develop shock need thorough head-to-toe assessments. Which of the following clinical manifestations would alert the health care provider that the client may be developing ischemia associated with gastrointestinal redistribution of blood flow? A) Gastric bleeding B) Nausea and vomiting C) Irritable bowel syndrome D) Copious high-volume diarrhea

A Feedback: In shock, there is widespread constriction of blood vessels that supply the gastrointestinal tract, causing a redistribution of blood flow that severely diminishes mucosal perfusion. Bleeding is a common symptom of gastrointestinal ulceration caused by shock, with onset usually within 2 to 10 days after the original insult. Nausea is unrelated to ischemic damage; irritable bowel syndrome is stress related. With ischemia, the bowel ceases to function, causing a lack of peristalsis and no fecal output.

A client is experiencing bladder hyperactivity. The nurse should be prepared to educate the client about which of the following medications that may be injected to help decrease the bladder hyperactivity? A) Capsaicin, a specific C-fiber afferent neurotoxin B) Botulinum toxin type A C) Oxybutynin, an antimuscarinic agent D) Urecholine, a cholinergic agonist

A Feedback: Intravesical injection of medications, such as capsaicin and resiniferatoxin, that are specific C-fiber afferent neurotoxins may be used to decrease bladder hyperactivity. Botulinum toxin type A is used to produce paralysis of striated muscles of the external sphincter. Oxybutynin, an antimuscarinic agent, will decrease detrusor muscle tone. Urecholine, a cholinergic agonist, stimulates parasympathetic receptors to increase bladder tone.

A drug abuser was found unconscious after shooting up heroin 2 days prior. Because of the pressure placed on the hip and arm, the client has developed rhabdomyolysis. The nurse knows this can: A) Obstruct the renal tubules with myoglobin and damage tubular cells B) Be cured by administering an anticoagulant immediately C) Cause the kidney to develop renal stones due to stasis D) Cause compartment syndrome in the lower extremities

A Feedback: Myoglobin normally is not found in the serum or urine. It has a low molecular weight; if it escapes into the circulation, it is rapidly filtered in the glomerulus. A life-threatening condition known as rhabdomyolysis occurs when increasing myoglobinuria levels cause myoglobin to precipitate in the renal tubules, leading to obstruction and damage to surrounding tubular cells. Myoglobinuria most commonly results from muscle trauma but may result from exertion, hyperthermia, sepsis, prolonged seizures, and alcoholism or drug abuse. Rhabdomyolysis is not cured with anticoagulation administration nor does it cause kidney stones. Compartment syndrome occurs when there is insufficient blood supply to muscles and nerves due to increased pressure within one of the body's compartments.

While assessing a peritoneal dialysis client in his or her home, the nurse notes that the fluid draining from the abdomen is cloudy, is white in color, and contains a strong odor. The nurse suspects this client has developed a serious complication known as: A) Peritonitis B) Bowel perforation C) Too much sugar in the dialysis solution D) Bladder erosion

A Feedback: Potential problems with peritoneal dialysis include infection, catheter malfunction, dehydration, hyperglycemia, and hernia. Bowel perforation can occur, but the fluid would be stool colored. The client may develop hyperglycemia; however, this will not cause the fluid to be cloudy. If bladder erosion had occurred, the fluid would look like urine and not be cloudy and white.

22-14. A 51-year-old man has been diagnosed with chronic bronchitis after a long history of recurrent coughing. Which of the man's following statements demonstrates a sound understanding of his new diagnosis? A) "If I had quit smoking earlier than I did, I think I could have avoided getting bronchitis." B) "I'm pretty sure that I first caught bronchitis from the person who has the cubicle next to mine at work." C) "I read on the Internet that I might have got bronchitis because I was born with an enzyme deficiency." D) "I think that I probably could have prevented this if I had got in the habit of exercising more when I was younger."

A Feedback: Smoking is frequently implicated in the etiology of chronic bronchitis. Infections do not typically initiate the disease, and exercise is not noted to have preventative value. Enzyme deficiency is associated with emphysema, but not bronchitis.

A bladder cancer client asks the nurse, "What did the doctor mean by intravesicular chemotherapy? Am I going to lose all my hair and have to do for treatments over months and months?" The best response would be: A) "This is when they put the chemotherapy directly into the bladder to kill any cancer cells." B) "They will take you to radiology and inject some chemotherapy through your abdomen into your bladder." C) "The doctor will place a scope up your urethra, into the bladder, and burn the lining of the bladder with a laser and then inject some tuberculosis bacillus into the lining." D) "This is when they use a CyberKnife to cut off any lesions and then inject chemotherapy into the remaining portion of the bladder."

A Feedback: Surgical treatment of superficial bladder cancer is often followed by intravesicular chemotherapy or immunotherapy, a procedure in which the therapeutic agent is directly instilled into the bladder. None of the other responses describe this procedure. The chemotherapy drug is not injected through the abdomen into the bladder. BCG is instilled into the bladder to elicit an inflammatory response that can kill the tumor. A CyberKnife is used with the brain, not the bladder.

The most recent assessment of a client with a diagnosis of type 1 diabetes indicates a heightened risk of diabetic nephropathy. Which of the following assessment findings is most suggestive of this increased risk? A) Microalbuminuria B) Hematuria C) Orthostatic hypotension D) Diabetic retinopathy

A Feedback: The increased glomerular filtration rate (GFR) that occurs in persons with early alterations in renal function is associated with microalbuminuria, which is an important predictor of future diabetic nephropathies. Hematuria is not directly suggestive of diabetic nephropathy, although it is a highly significant assessment finding. Orthostatic hypotension and diabetic retinopathy are not direct indicators of diabetic nephropathy.

Clients with CKD are at risk for demineralization of their bones since they are no longer able to: A) Transform vitamin D to its active form B) Excrete bicarbonate effectively C) Stimulate bone osteoclastic production D) Synthesize erythropoietin

A Feedback: The kidneys aid in calcium metabolism by activating vitamin D, after it is chemically converted by the liver. Bicarbonate buffering is unrelated to activation of vitamin D. Bone marrow is stimulated by the synthesis of erythropoietin to form red blood cells, which is unrelated to calcium levels.

A newly diagnosed paraplegic client who suffered an automobile accident appears to have control of bladder emptying. The health care provider explains this process to the client/family stating, "This function is allowing the motor component of the neural reflex to assist with bladder emptying and is primarily controlled by the: A) Parasympathetic division of the ANS." B) Sympathetic division of the ANS." C) Somatic nervous system." D) Hypogastric nervous system."

A Feedback: The motor component of the neural reflex to assist with bladder emptying is primarily controlled by the parasympathetic division of the ANS, and the relaxation and storage functions of the bladder are controlled by the sympathetic division. The somatic nervous system innervates the skeletal muscles of the external sphincter and the pelvic floor muscles that together control the outflow of urine. The afferent input from the bladder and urethra is carried to the CNS by fibers that travel with PS, somatic, and sympathetic (hypogastric) nerves.

22-22. A young, male child is born with severe respiratory failure. Over the course of months, the parents note his body looks swollen. They ask, "Is our baby's kidneys not working right? Why is he so swollen?" The nurse bases his or her reply on which of the following physiological principles? A) "The right side of his heart (cor pulmonale) is not pumping effectively. Blood is backlogging in his body, which is why he is so swollen." B) "We just need to call the physician and ask him to give you a prescription for more water pills." C) "Once we get his oxygenation level back to normal, then maybe his kidneys will receive enough oxygenated blood to filter better." D) "This happens when he has so many secretions in his lungs. Maybe we should try some expectorant to thin his secretions so he can cough them out."

A Feedback: The term cor pulmonale refers to right heart failure resulting from primary lung disease or pulmonary hypertension. The increased pressures and work result in hypertrophy and eventual failure of the right ventricle. The manifestations of cor pulmonale include signs of right-sided heart failure, which include venous congestion, peripheral edema, shortness of breath, and a productive cough, which becomes worse during periods of heart failure. None of the other statements are applicable to these parents' questions.

While teaching a client with new-onset right- sided heart failure, the nurse should educate the client to monitor for fluid accumulation by: A)Weighing every day at the same time with same type of clothing B) Measuring all of the client's urine output daily to check for a decrease in output C) Listening to the breath sound with a stethoscope every morning D) Take blood pressure daily and call doctor if it is decreased

A Feedback: When the right heart fails, a damming back of blood occurs, leading to its accumulation in the systemic venous system, causing an increase in right atrial, right ventricular end- diastolic, and systemic venous pressures. The accumulation of fluid (edema) is evidenced by a gain in weight (i.e., 1 pint of accumulated fluid results in a 1-pound weight gain). Shortness of breath due to congestion of the pulmonary circulation is one of the major manifestations of left-sided heart failure. It is unrealistic to expect clients to listen to their own breath sounds. BP measurement could be an intervention; however, it is not a primary indicator of edema from right-sided heart failure. With impairment of left heart function, there is a decrease in cardiac output, with resulting decreased renal perfusion and output.

A woman has sought care because of recurrent urinary tract infections, which have been increasing in both frequency and severity. The health care worker should explain which of the following physiological factors to the client that is likely contributing to recurrent UTIs? A) Reflux flow of urine that can occur from coughing or sneezing B) Fluctuations in urine pH related to beverage consumption C) Urethral trauma that occurs during sexual intercourse D) Inadequate intake of water

A feedback A phenomenon called urethrovesical reflux occurs when urine from the urethra moves into the bladder. In women, urethrovesical reflux can occur during activities such as coughing or squatting, in which an increased intra-abdominal pressure causes the urine to be squeezed into the urethra and then to flow back into the bladder as the pressure decreases. Reflux flow of urine is a significant risk factor for UTIs in general and for recurrent UTIs in particular. Fluctuations in urine pH are not noted to contribute to recurrent UTIs. Urethral trauma and inadequate fluid intake may contribute to the development or prolonging of UTIs, but these risk factors are less significant than the presence of urine reflux.

A client has recently undergone successful extracorporeal shock wave lithotripsy (ESWL) for the treatment of renal calculi. Which of the following measures should the client integrate into his lifestyle to reduce the risk of recurrence? A) Increased fluid intake and dietary changes B) Weight loss and blood pressure control C) Regular random blood glucose testing D) Increased physical activity and use of over- the-counter diuretics

A feedback Depending on the type of stone that was present, many clients benefit from increased fluid intake and changes in diet. Weight loss, blood sugar and pressure control, and exercise are not central preventative measures. It would likely be inappropriate to recommend the use of over-the-counter diuretics as a preventative measure.

A client with a diagnosis of chronic kidney disease (CKD) may require the administration of which of the following drugs to treat coexisting conditions that carry a high mortality? A) Antihypertensive medications B) Antiarrhythmic medications C) Opioid analgesics D) Nonsteroidal anti-inflammatory drugs (NSAIDs)

A Feedback: Hypertension is a common result of CKD, and the mechanisms that produce hypertension in CKD include increased vascular volume, elevation of peripheral vascular resistance, decreased levels of renal vasodilator prostaglandins, and increased activity of the renin-angiotensin-aldosterone system. NSAIDs, opioids, and antiarrhythmics are not as frequently indicated for the treatment of CKD

The most damaging effects of urinary obstruction are the result unrelieved obstruction of urine outflow and: A) Urinary stasis B) Concentrated urine C) Kidney hyperplasia D) Renal hypertension

A Feedback: The most damaging effects of urinary obstruction are stasis of urine, which predisposes to infection and stone formation, and unrelieved obstruction of urine outflow. Most commonly, the person has pain, signs, and symptoms of urinary tract infection (UTI) and manifestations of renal dysfunction, such as an impaired ability to concentrate urine. Progressive atrophy of the kidney is caused by obstruction of the outflow of urine. Hypertension is an occasional complication of urinary tract obstruction, since urine flow is obstructed rather than renal blood flow.

Which of the following signs and symptoms in a 2-year-old child should prompt assessment for a urinary tract infection? A) Unexplained fever and anorexia B) Decreased urine output and irritability C) Production of concentrated urine and recurrent nausea D) Frank hematuria

A feedback Although all of the cited symptoms warrant further assessment and follow-up, the presence of fever and anorexia is typical of UTIs in toddlers.

A client has a diagnosis of chronic renal failure secondary to diabetic nephropathy. Which of the following hematologic changes may result from this client's kidney disorder? A) Anemia B) Leukocytosis C) Thrombocytopenia D) Leukopenia

A feedback Erythropoietin is a polypeptide hormone that regulates the differentiation of RBCs in the bone marrow. Between 89% and 95% of erythropoietin is formed in the kidneys. The synthesis of erythropoietin is stimulated by tissue hypoxia, which may be brought about by anemia, residence at high altitudes, or impaired oxygenation of tissues due to cardiac or pulmonary disease. Persons with chronic kidney disease often are anemic because of an inability of the kidneys to produce erythropoietin. Changes in platelet or white blood cell levels are not likely to result directly from renal failure.

A client with a recent diagnosis of renal failure who will require hemodialysis is being educated in the dietary management of the disease. Which of the client's following statements shows an accurate understanding of this component of treatment? A) "I've made a list of high-phosphate foods, so that I can try to avoid them." B) "I'm making a point of trying to eat lots of bananas and other food rich in potassium." C) "I'm going to try to maintain a high-fiber, low-carbohydrate diet." D) "I don't think I've been drinking enough, so I want to include 8 to 10 glasses of water each day."

A feedback Persons with chronic kidney disease (CKD) are usually encouraged to limit their dietary phosphorus as a means of preventing secondary hyperparathyroidism, renal osteodystrophy, and metastatic calcification. Excessive potassium and fluids are likely contraindicated in kidney disease individuals require hemodialysis. The amount of dietary fiber intake is not a priority when looking at primary needs of a CKD patient's food intake.

A client is admitted with worsening heart failure. The client is complaining about having to urinate frequently. The nurse knows that the physiology behind the body's response to decrease vascular volume by increasing urine output is due to: A) Release of atrial natriuretic peptide (ANP) from overstretched atria B) Renin secretion, resulting in angiotensin II formation C) Reabsorption of potassium from the proximal tubule D) Aldosterone secretion by the adrenal gland, which inhibits sodium absorption

A feedback ANP is believed to play an important role in salt and water excretion by the kidney. It is synthesized by muscle cells in the atria of the heart and released when the atria are stretched. Increased levels of this peptide directly inhibit the reabsorption of sodium and water in the renal tubules. ANP also inhibits renin secretion and therefore angiotensin II formation, which in turn reduces reabsorption of sodium. The decrease in sodium reabsorption increases urine output and helps return blood volume to normal. ANP levels, which become elevated when the atria are stretched in HF, help to decrease vascular volume by increasing urine output. Potassium reabsorption is not responsible for water excretion. Aldosterone secretion by the adrenal gland functions in the regulation of sodium and potassium elimination by the principal cells in the distal and collecting tubules.

The most common causes of left-sided heart failure include: A) Acute myocardial infarction B) Chronic pulmonary disease C) Impaired renal blood flow D) Tricuspid valve regurgitation

A feedback: The most common causes of left-sided heart failure are acute myocardial infarction and hypertension. Acute or chronic pulmonary disease can cause right heart failure, referred to as cor pulmonale. The causes of right-sided heart failure include stenosis or regurgitation of the tricuspid or pulmonic valves, right ventricular infarction, and cardiomyopathy. Manifestations (rather than causes) of heart failure reflect the physiologic effects of the impaired pumping ability of the heart, including decreased renal blood flow.

16. A patient with a history of chronic obstructive pulmonary disease (COPD) is undergoing pulmonary function testing. Which of the following instructions should the technician provide in order to determine the patient's forced vital capacity (FVC)? A) "I'll ask you to breathe in as deep as you can, and then blow out as much of that air as possible." B) "I'd like you to take a deep breath, and then blow out as much air as you can during one second." C) "I want you to breathe as normally as possible and I'm going to measure how much air goes in and out with each breath." D) "Breathe normally, and then exhale as much as you possibly can when I tell you."

A) "I'll ask you to breathe in as deep as you can, and then blow out as much of that air as possible."

A 29-year-old woman who considers herself active and health conscious is surprised to have been diagnosed with preeclampsia-eclampsia in her second trimester. What should her care provider teach her about this change in her health status? A) "We don't really understand why some women get high blood pressure when they're pregnant." B) "This is likely a result of your nervous system getting overstimulated by pregnancy." C) "Hypertension is a common result of all the hormonal changes that happen during pregnancy." D) "Even though you're a healthy person, it could be that you have an underlying heart condition."

A) "We don't really understand why some women get high blood pressure when they're pregnant."

Which of the following is a nonmodifiable risk factor for the development of primary hypertension? A) African American race B) High salt intake C) Male gender D) Obesity

A) African American race

Serology testing includes the measurement of which of the following? A) Antibody titers B) Culture growth C) Direct antigens D) DNA sequencing

A) Antibody titers

Coronary artery bypass grafting (CABG) is a relevant treatment modality for which of the following disorders of cardiac function? A) Atherosclerosis with history of MI B) Pericardial effusion and cardiac tamponade C) Dilated cardiomyopathies D) Aortic valve regurgitation and aortic stenosis

A) Atherosclerosis with history of MI

A patient's primary care provider has added 20 mg of Lasix (furosemide) to his medication regimen to treat his primary hypertension. How does this diuretic achieve its therapeutic effect? A) By decreasing vascular volume by increasing sodium and water excretion B) By blocking the release of antidiuretic hormone from the posterior pituitary C) By inhibiting the conversion of angiotensin I to angiotensin II. D) By inhibiting the movement of calcium into arterial smooth muscle cells

A) By decreasing vascular volume by increasing sodium and water excretion

A patient is receiving homecare for the treatment of a wound on the inside of her lower leg which is 3 cm in diameter with a yellow wound bed and clear exudate. Assessment of the patient's legs reveals edema and a darkened pigmentation over the ankles and shins of both legs. What is this patient's most likely diagnosis? A) Chronic venous insufficiency B) Deep vein thrombosis C) Varicose veins D) Peripheral arterial disease

A) Chronic venous insufficiency

A patient has recently received a pneumococcal vaccine and the patient's B cells are consequently producing antibodies. Which of the following cells may enhance this production of antibodies? A) Helper T cells B) Regulatory T cells C) Cytotoxic T cells D) Natural killer cells

A) Helper T cells

Histocompatability molecules are of primary importance to which of the following aspects of immunity? A) Induction of T-cell immunity B) T-cell maturation C) NK cell activation D) Phagocytosis by neutrophils

A) Induction of T-cell immunity

An elderly female patient with complaints of increasing fatigue has been diagnosed with aortic stenosis, a disease which her primary care provider believes may have been long- standing. Which of the following compensatory mechanisms has most likely maintained the woman's ejection fraction until recently? A) Left ventricular hypertrophy B) Increased blood pressure C) Increased heart rate and stroke volume D) Aortic dilation

A) Left ventricular hypertrophy

Preload represents the volume work of the heart and is largely determined by: A) venous blood return. B) vascular resistance. C) force of contraction. D) ventricular emptying.

Ans: A) venous blood return.

When a client with a history of asthma takes a walk outside on a windy day with high pollen counts, she may experience an asthma attack, resulting in an increase in respiration rate and wheezing. The body's response is likely related to which pathophysiological principle? A) Parasympathetic nervous system stimulation resulting in airway constriction B) Release of catecholamines causing blood vessel constriction C) Influx of macrophages to wall of the pollen, thereby stopping the attack D) Inhibition of glandular secretions, which causes build up of mucus in the lungs

A) Parasympathetic nervous system stimulation resulting in airway constriction Feedback: The parasympathetic (PS) fibers are excitatory neurons that respond to acetylcholine. Stimulation of the PSN is responsible for airway constriction, blood vessel dilation, and increased glandular secretion. The sympathetic nervous system (SNS), which responds to the catecholamines norepinephrine and epinephrine, produces airway dilation, blood vessel constriction, and inhibition of glandular secretions.

Prior to leaving on a backpacking trip to Southeast Asia, a college student has received a tetanus booster shot. This immunization confers protection by way of what immune process? A) Secondary humoral response B) Cell-mediated immune response C) Primary humoral response D) Innate immunity

A) Secondary humoral response

The plaques in a patient's coronary arteries are plentiful and most have small- to moderate-sized lipid cores with thick fibrous caps. This form of atherosclerosis is most closely associated with which of the following diagnoses? A) Stable angina B) Non-ST-segment elevation MI C) ST-Segment elevation MI D) Unstable angina

A) Stable angina

A patient's cell-mediated immune response has resulted in the release of regulator T cells. These cells will perform which of the following roles? A) Suppressing the immune response to limit proliferation of potentially harmful lymphocytes B) Presenting antigens to B cells to facilitate the production of antibodies C) Differentiating into subpopulations of helper T cells D) Destroying target cells by releasing cytolytic enzymes and other toxins

A) Suppressing the immune response to limit proliferation of potentially harmful lymphocytes

Which of the following traits is characteristic of saprophytes? A) They derive energy from decaying organic matter. B) They are beneficial components of human microflora. C) They have RNA or DNA, but never both. D) They are capable of spore production.

A) They derive energy from decaying organic matter.

The carbon dioxide content in the blood affects the regulation of ventilation through its effect on the ___________ of the brain. A) cerebrospinal fluid pH B) peripheral chemoreceptors C) afferent impulse regulation D) motor and premotor cortex

A) cerebrospinal fluid pH

Bronchial circulation differs from the pulmonary circulation by providing blood for the: A) conducting airways. B) alveolar gas exchange. C) mediastinum and pleural space. D) intrapulmonary pressure balance.

A) conducting airways.

Chronic stable angina, associated with inadequate blood flow to meet the metabolic demands of the myocardium, is caused by: A) fixed coronary obstruction. B) increased collateral circulation. C) intermittent vessel vasospasms. D) excessive endothelial relaxing factors.

A) fixed coronary obstruction.

A major cause of secondary hyperlipoproteinemia is _______, which increases the production of VLDL and conversion to LDL. A) high-calorie diet B) diabetes mellitus C) bile-binding resin D) cholesterol ingestion

A) high-calorie diet

Dendritic cells, found in skin tissues and lymphoid tissues, are important for: A) initiation of adaptive immunity. B) deep-tissue phagocytosis. C) disposal of dead cells. D) delaying inflammation.

A) initiation of adaptive immunity.

During the latent period before antibodies are detected in the humoral immune response, B cells differentiate into ________ cells. A) plasma B) cytotoxic C) stem D) helper

A) plasma

A child with asthma is walking outside and develops a bronchospasm. The school nurse knows this bronchospasm has what effects on bronchioles with airflow? Select all that apply. A) Narrowing of bronchioles B) Impairs airflow C) Exerts tension on the bronchiole walls D) Causes atelectasis in posterior lung segments E) Inhibits the filtering of dust particles

A, B Feedback: Bronchospasm, or contraction of these muscles, causes narrowing of the bronchioles and impairs airflow. Bronchospasm does not exert tension on the walls of the bronchi, cause atelectasis, or inhibit filtering of dust particles.

Which of the following clients are likely experiencing a shift to the right in the dissociation curve? Select all that apply. A) A client with respiratory influenza with a temperature of 102.6°F B) A COPD client with pneumonia with blood gas pH level of 7.31 C) A renal failure client with admitting hemoglobin level of 8.0 mg/dL D) A client having a panic attack with respiratory rate of 36 and marked decreased PCO2 levels E) A client who fell in an icy pond with admitting core body temperature of 94.1°F

A, B, C Feedback: A shift to the right indicates that the affinity of hemoglobin for oxygen is decreased and the PO2 that is available to the tissues at any given level of hemoglobin saturation is increased. It usually is caused by conditions that produce an increase in tissue metabolism, such as fever or acidosis, or by an increase in PCO2. High altitude and conditions such as pulmonary insufficiency, heart failure, and severe anemia also cause the oxygen dissociation curve to shift to the right. A shift to the left indicates that the affinity of hemoglobin for oxygen is increased and the PO2 that is available to the tissues at any given level of hemoglobin saturation is decreased. It occurs in situations associated with a decrease in tissue metabolism, such as alkalosis, decreased body temperature, and decreased PCO2 levels.

Following an episode of strep throat, the school nurse notices the fourth grade child has not recovered from this illness a week later. Upon further investigation, the nurse notices that the child has developed water retention. Which of the following assessments support this conclusion? Select all that apply. A) Periorbital edema B) BP 100/70 C) Swelling of the hands and fingers D) Vomiting after intake of any solid food E) Dizziness and right ear pain

A,B Feedback: Generalized edema, a hallmark of nephrosis, results from salt and water retention and a decrease in plasma colloid osmotic pressure due to loss of albumin in the urine. Glomerulonephritis is characterized by sodium and water retention that causes edema, particularly of the face and hands. Fluid retention usually results in an elevated BP, not a normal one. Vomiting and dizziness are not associated with this diagnosis.

The client with emphysema should be educated about changes in lung tissue that may include which of the following changes? Select all that apply. A) Loss of elastic recoil B) Difficulty exhaling due to inability to recoil C) Increased lung compliance D) Increased permeability to water E) Stiff elastin fibers

A, B, C Feedback: The elastic properties of the lung involve at least three basic components: distensibility, stiffness, and elastic recoil. Distensibility is the ease with which the lungs can be inflated. Stiffness is defined as the resistance to stretch or inflation. In lung diseases such as interstitial lung disease and pulmonary fibrosis, the lungs become stiff and noncompliant as the elastin fibers are replaced with the collagen fibers of scar tissue. Overstretching lung tissues, as occurs with emphysema, causes the elastic components of the lung to lose their recoil, making the lung more compliant and easier to inflate but more difficult to deflate because of its inability to recoil.

The client has had prolonged urethral outlet obstruction. The nurse knows that physiologically, the client may likely develop small pockets of mucosal tissue, called cellulae, which can ultimately cause: Select all that apply. A) Infections due to stasis B) Backpressure on the ureters C) Development of hydroureters D) Sphincter dystonia

A, B, C Feedback: With continued outflow obstruction, this smooth surface is replaced with coarsely woven structures (i.e., hypertrophied smooth muscle fibers) called trabeculae. Small pockets of mucosal tissue, called cellulae, commonly develop between the trabecular ridges. These pockets form diverticula when they extend between the actual fibers of the bladder muscle. Because the diverticula have no muscle, they are unable to contract and expel their urine into the bladder, and secondary infections caused by stasis are common. Along with hypertrophy of the bladder wall, there is hypertrophy of the trigone area and the interureteric ridge, which is located between the two ureters. This causes backpressure on the ureters, the development of hydroureters and, eventually, kidney damage. Detrusor-sphincter dyssynergia is uncoordinated activity that causes overdistention. Sphincter dystonia is a cause of incontinence. Interstitial cystitis is a condition of increased sensitivity to bladder pressure, loss of bladder elasticity, and severe urgency unrelated to outlet obstruction.

The shortness of breath and cyanosis that occur in clients experiencing acute heart failure syndrome are primarily caused by: Select all that apply. A) Accumulation of fluid in the alveoli and airways B) Lung stiffness C) Worsening renal failure D) Myocardial muscle necrosis E) Impaired gas exchange

A, B, E Feedback: Acute pulmonary edema is the most dramatic symptom of AHFS. It is a life-threatening condition in which capillary fluid moves into the alveoli. The accumulated fluid in the alveoli and airways causes lung stiffness, makes lung expansion more difficult, and impairs the gas exchange function of the lung. With the decreased ability of the lungs to oxygenate the blood, the hemoglobin leaves the pulmonary circulation without being fully oxygenated, resulting in shortness of breath and cyanosis. Worsening renal failure and MI may cause volume overload but are more likely secondary causes of chronic heart failure.

When explaining urinalysis results that show the presence of cast cells, the nurse informs the client that casts cells develop when the client has: Select all that apply. A) A high protein concentration of the urine B) An elevated urine pH C) High urine osmolality D) More than one bacteria present in the urine

A, C Feedback: Casts are molds of the distal nephrons' lumen. A gel-like substance (Tamm-Horsfall mucoprotein) is formed in the tubular epithelium and is the major protein constituent of urinary casts. Casts composed of this gel but devoid of cells are called hyaline casts. These casts develop when the protein concentration of the urine is high, urine osmolality is high, and urine pH is low.

22-13. While lecturing on COPD, the instructor mentions emphysema. The instructor asks the students, "If the client is a smoker, explain the physiology behind cellular changes occurring in the lung which allow destruction of the alveoli." Which students have an accurate response? Select all that apply. A) "Antiprotease production and release is not adequate to neutralize the excess protease production." B) "The capillary beds can no longer bring the cells to the lung to fight off the infection since they are blocked by fatty plaque." C) "There is α1-antitrypsin deficiency, so this enzyme can't protect the lung from damage." D) "The alveolar tissue is being digested by enzymes; therefore, they can't grow back to restore normal ventilation."

A, C Feedback: Emphysema is characterized by a loss of lung elasticity and abnormal enlargement of the air spaces distal to the terminal bronchioles, with destruction of the alveolar walls. One of the recognized causes of emphysema is an inherited deficiency of α1-antitrypsin, an antiprotease enzyme that protects the lung from injury. Cigarette smoke stimulates the movement of inflammatory cells into the lungs, resulting in increased release of elastase and other proteases. In smokers in whom COPD develops, antiprotease production and release may be inadequate to neutralize the excess protease production such that the process of elastic tissue destruction goes unchecked.

When acute tubular necrosis (ATN) is suspected, the nurse will likely see which of the following laboratory findings on the urinalysis report? Select all that apply. A) Protein B) Glucose C) Red blood cells D) Sodium excess E) Cast cells

A, C, E Feedback: Nephron damage allows the larger protein cells to pass through the membrane and into the urine (normally, urine has very few proteins present). Further diagnostic information that can be obtained from the urinalysis includes hemoglobinuria (blood in the urine) and casts or crystals in the urine. Glucosuria in the urine is an indirect indication of extreme hyperglycemia, often unrelated to renal disease. Urine sodium concentration is maintained with prerenal azotemia; urine sodium decreases with renal tubule damage. Urine calcium is not diagnostic for ATN.

While studying for renal test in pathophysiology class, a student helps a peer by reviewing facts about the cells of the proximal tubule. Which of the following functions should they include in this discussion? Select all that apply. A) Aids in reabsorption B) Location for high-pressure capillary filtration C) Rich in mitochondria D) Plays a role in the medullary collecting tubule E) Supports active transport processes

A, C, E Feedback: The cells of the proximal tubule have a fine, villous structure that increases the surface area for reabsorption; they also are rich in mitochondria, which support active transport processes. The epithelial layer thins in segments of the loop of Henle and has few mitochondria, indicating minimal metabolic activity and reabsorptive function.

At the cellular level, cardiac muscle cells respond to an increase in ventricular volume to the point of overload by: Select all that apply. A) Elongating the cardiac muscle cells B) Thickening of the individual myocytes C) Replicating the myofibrils D) Decreasing the ventricular wall thickness E) Symmetrically widening and lengthening the hypertrophy

A, D Feedback: At the cellular level, cardiac muscle cells respond to stimuli from stress placed on the ventricular wall by pressure and volume overload by initiating several different processes that lead to hypertrophy. With ventricular volume overload, the increase in wall stress leads to replication of myofibrils in series, elongation of the cardiac muscle cells, and eccentric hypertrophy. Eccentric hypertrophy leads to a decrease in ventricular wall thickness or thinning of the wall with an increase in diastolic volume and wall tension. Production of a symmetric hypertrophy occurs with a proportionate increase in muscle length and width, as occurs in athletes; concentric hypertrophy with an increase in wall thickness, as occurs in hypertension; and eccentric hypertrophy with a disproportionate increase in muscle length, as occurs in dilated cardiomyopathy. When the primary stimulus for hypertrophy is pressure overload, the increase in wall stress leads to parallel replication of myofibrils, thickening of the individual myocytes, and concentric hypertrophy. Concentric hypertrophy may preserve systolic function for a time, but eventually the work performed by the ventricle exceeds the vascular reserve, predisposing to ischemia.

From the following clients, who are at high risk for developing heart failure as a result of diastolic dysfunction? Select all that apply. A) A 48-year-old client with uncontrolled hypertension B) A marathon runner with history of chronic bradycardia whose pulse rate is 46 C) A 57-year-old client with history of ischemic heart disease D) A 70-year-old with enlarged left ventricle due to myocardial hypertrophy

A, D Feedback: Conditions that reduce the heart's ability to adequately fill during diastole, such as myocardial hypertrophy and tachycardia, can lead to heart failure. Hypertension remains the leading cause of diastolic dysfunction. Ischemic heart disease is associated with systolic heart failure, or impaired contractile performance. It is normal for athletes, like marathon runners, to have slow pulses.

If a CKD client is developing uremic encephalopathy, the earliest manifestations may include: Select all that apply. A) Decreased alertness B) Delirium and hallucinations C) New-onset seizures D) Diminished awareness

A, D Feedback: Reductions in alertness and awareness are the earliest and most significant indications of uremic encephalopathy. Late in the disease process, the client may develop delirium, coma, and seizures.

Nitric oxide, a vasodilator produced by the vascular endothelium, is important in renal control by: Select all that apply. A) Preventing excessive vasoconstriction B) Regulating renal blood flow C) Inhibiting prostaglandin synthesis D) Allowing normal excretion of sodium and water

A, D feedback: Nitric oxide, a vasodilator produced by the vascular endothelium, appears to be important in preventing excessive vasoconstriction of renal blood vessels and allowing normal excretion of sodium and water. Prostaglandins do not appear to play a major role in regulating renal blood flow. Some medications like aspirin and NSAIDs inhibit prostaglandin synthesis.

Electrical burns over a large surface area of a client's body have resulted in hypovolemic shock after the loss of large amounts of blood and plasma. Following physical assessment, which findings lead the nurse to believe the client's body is compensating for this fluid loss? Select all that apply. A) Increased heart rate B) Vasodilation with warm extremities C) Diuresis with output of 100 mL/hour D) The client complaining of extreme thirst E) Deep, rapid respirations

A, D, E Feedback: Compensatory mechanisms in hypovolemic shock include increased heart rate, peripheral vasoconstriction, and fluid and sodium retention in order to preserve vascular volume. Urine output decreases very quickly in hypovolemic shock. Thirst is an early symptom in hypovolemic shock. As shock progresses, the respirations become rapid and deep to compensate for the increased production of acid and decreased availability of oxygen.

1. Which of the following clients are more than likely experiencing impairment of the mucociliary blank with cilia dysfunction? Select all that apply. A) A smoker who smokes 2 packs of cigarettes/ day and currently hospitalized with pneumonia B) A middle-aged diabetic client with bilateral neuropathy C) A military guard person stationed in Germany D) A mountain skier who spends all day outside teaching ski lessons E) A nursing home client diagnosed with H1N1 influenza with fever of 102°F

A, D, E Feedback: The mucociliary blanket protects the respiratory system by entrapping foreign particles in mucus; then cilia move the mucus with trapped particles upward toward the oropharynx to be coughed out or swallowed. The function of the cilia in clearing the lower airways and alveoli is optimal at normal oxygen levels and is impaired in situations of low and high oxygen levels. It is also impaired by drying conditions, such as breathing heated but unhumidified indoor air during the winter months. Cigarette smoking slows down or paralyzes the motility of the cilia. This slowing allows the residue from tobacco smoke, dust, and other particles to accumulate in the lungs, decreasing the efficiency of this pulmonary defense system. During fever, the water vapor in the lungs increases, causing more water to be lost through the respiratory tract. In addition, fever usually is accompanied by an increase in respiratory rate so that more air passes through the airways, withdrawing moisture from its mucosal surface. As a result, respiratory secretions thicken, preventing free movement of the cilia and impairing the protective function of the mucociliary defense system.

As nitrogenous wastes increase in the blood, the CKD client may exhibit which of the following clinical manifestations? Select all that apply. A) Numbness in lower extremities B) Photophobia C) Extremely low platelet counts D) Restless leg syndrome E) Pruritis

A, D, E Feedback: The uremic state is characterized by signs and symptoms of altered neuromuscular function (e.g., fatigue, peripheral neuropathy, restless leg syndrome, sleep disturbances, uremic encephalopathy); gastrointestinal disturbances such as anorexia and nausea; white blood cell and immune dysfunction, and dermatologic manifestations such as pruritus. Photophobia and thrombocytopenia are usually not associated with CKD.

A client has been prescribed hydrochlorothiazide, a thiazide diuretic, to control high blood pressure. While educating the client about the actions of the medication, the nurse will mention which of the following actions? Select all that apply. A) Blocks Na+ reabsorption in distal tubules B) Establishes a high concentration of osmotically active particles C) Increases active reabsorption of Ca++ into the blood D) Juxtamedullary vasoconstriction, which controls water movement E) Establishes a high concentration of K+ within the cell

A,C Feedback The thiazide diuretics, which are widely used to treat disorders such as hypertension, exert their action by blocking sodium reabsorption in this segment of the renal tubules, while enhancing the active reabsorption of calcium into the blood via the calcium-sodium exchange transport mechanism. For this reason, thiazide diuretics have proved useful in reducing the incidence of calcium kidney stones in persons with hypercalciuria. ATPase pump maintains a low sodium concentration inside the cell by moving sodium down its concentration into the cell through special sodium channels. The pump also establishes a high concentration of potassium within the cell, causing it to diffuse down its concentration gradient across the luminal membrane into the tubular fluid.

A chronic kidney disease client who has renal osteodystrophy should be assessed for which of the following complications? Select all that apply. A) Muscle weakness B) Kidney stones C) Bone pain D) Stress fractures E) Urosepsis

A,C, D Feedback: Both types of renal osteodystrophy are manifested by abnormal absorption and defective bone remodeling. Renal osteodystrophy is typically accompanied by reductions in bone mass, alterations in bone microstructure, bone pain, and skeletal fracture. There are changes in bone turnover, mineralization, and bone volume, accompanied by bone pain and muscle weakness, risk of fractures, and other skeletal complications. Kidney stones and urosepsis are not associated with renal osteodystrophy.

If a client with a kidney stone has the "classic" ureteral colic, the client will describe his pain as: Select all that apply. A) Acute, intermittent B) Diffuse over the entire lower back and legs C) Excruciating D) In the flank and upper outer quadrant of the abdomen

A,C,D feedback The symptoms of renal colic are caused by stones 1 to 5 mm in diameter that can move into the ureter and obstruct flow. Classic ureteral colic is manifested by acute, intermittent, and excruciating pain in the flank and upper outer quadrant of the abdomen on the affected side. The pain may radiate to the lower abdominal quadrant, bladder area, perineum, or scrotum in the man. The pain is usually not described as diffuse and over the entire low back and legs.

A client arrives in the emergency department suffering a traumatic brain injury as a result of a car accident. While assessing this client, the nurse notices the client has an irregular breathing pattern consisting of prolonged inspiratory gasps interrupted by expiratory efforts. The underlying physiological principle for these signs would include: A) Damage has occurred at the connection between the pneumotaxic and apneustic centers. B) The client's occipital lobe is no longer functioning. C) The client must have a leak in the ventricles resulting in a decrease in spinal fluid. D) The nerves innervating the lungs have been severed in the accident.

A. Damage has occurred at the connection between the pneumotaxic and apneustic centers. Feedback: Brain injury, which damages the connections between the pneumotaxic and apneustic centers, results in an irregular breathing pattern that consists of prolonged inspiratory gasps interrupted by expiratory efforts. If the occipital lobe was not functioning, the client would have no respiratory effort and require mechanical ventilation. Leaking of spinal fluid would not cause these respiratory signs. If nerves were severed to the lungs, the client would not be able in inflate/deflate the lungs with mechanical ventilation.

Which of the following would constitute a normal assessment finding in a neonate? A) Minimal or absent levels of IgA and IgM B) Absence of plasma cells in the lymph nodes and spleen C) Undetectable levels of all immunoglobulins D) Absence of mature B cells with normal T-cell levels and function

ANs: A feedback IgA and IgM levels are normally low in the neonate because these immunoglobulins do not cross the placental barrier. An absence of plasma cells in the lymph nodes and spleen accompanies common variable immunodeficiency. An absence of mature B cells with normal T-cell levels and function, as well as undetectable levels of all immunoglobulins, is a pathologic finding in X-linked agammaglobulinemia.

By definition, hypertension is systolic blood pressure of ____ mm Hg or higher or diastolic blood pressure of ____ mm Hg or higher. A) 129; 85 B) 138; 89 C) 140; 90 D) 155; 95

ANs: C) 140; 90

During ventricular systole, closure of the atrioventricular (AV) valves coincides with: A) atrial chamber filling. B) aortic valve opening. C) isovolumetric contraction. D) semilunar valves opening.

ANs: C) isovolumetric contraction.

A client has just been diagnosed with Graves disease. The primary care provider tells the client that this is due to the presence of autoantibodies to the TSH receptor. The client asks the nurse to explain this in simpler terms. Which response would be the best for this client? A) "Hyperthyroidism is basically an autoimmune disorder." B) "Basically your lymph node system has gone haywire." C) "Your body has loss its ability to delete autoreactive T cells in the thyroid." D) "The filtration system in your body is no longer working."

Ans: A Feedback: Several mechanisms are available to filter autoreactive B cells out of the B-cell population: clonal deletion of immature B cells in the bone marrow; deletion of autoreactive B cells in the spleen or lymph nodes; functional inactivation or anergy; and receptor editing, a process that changes the specificity of a B-cell receptor when autoantigen is encountered. There is increasing evidence that B-cell tolerance is predominantly due to help from T cells. Loss of self-tolerance with development of autoantibodies is characteristic of a number of autoimmune disorders. For example, hyperthyroidism in Graves disease is due to autoantibodies to the TSH receptor.

One of the principal mechanisms by which the heart compensates for increased workload is: A) Myocardial hypertrophy B) Sodium and water retention C) Endothelin vasoconstrictors D) Ventricular wall tension increase

Ans: A Feedback: The development of myocardial hypertrophy constitutes one of the principal mechanisms by which the heart compensates for an increase in workload. There are at least two types of endothelin receptors, and it is thought that the peptide may play a role in mediating noncompensatory pulmonary hypertension in persons with heart failure. One effect of a lowered cardiac output in heart failure is a noncompensatory reduction in renal blood flow and glomerular filtration rate, which leads to salt and water retention. Because increased wall tension increases myocardial oxygen requirements, it can produce noncompensatory ischemia and further impairment of cardiac function.

A patient has entered hypovolemic shock after massive blood loss in a car accident. Many of the patient's peripheral blood vessels have consequently collapsed. How does the Laplace law account for this pathophysiologic phenomenon? A) Blood pressure is no longer able to overcome vessel wall tension. B) Decreasing vessel radii have caused a decrease in blood pressure. C) Wall thickness of small vessels has decreased due to hypotension. D) Decreases in wall tension and blood pressure have caused a sudden increase in vessel radii.

Ans: A) Blood pressure is no longer able to overcome vessel wall tension.

Prions cause transmissible neurodegenerative diseases and are characterized by: A) a lack of reproductive capacity. B) hypermetabolism. C) enzyme production. D) chronic inflammation.

Ans: A) a lack of reproductive capacity.

Long-term autoregulation of local blood flow in the microcirculation is mediated by: A) collateral circulation. B) arteriovenous shunting. C) autonomic nervous system. D) metabolic needs of the tissues.

Ans: A) collateral circulation.

Turbulent blood flow can be caused by a number of factors, including: A) increased velocity. B) short vessel length. C) high blood viscosity. D) layering of blood cells.

Ans: A) increased velocity.

The difference between the end-diastolic and end-systolic volumes is the: A) stroke volume. B) cardiac output. C) ejection fraction. D) cardiac reserve.

Ans: A) stroke volume.

Which of the following diagnostic and assessment results support the diagnosis of chronic pyelonephritis? Select all that apply. A) Polyuria (excess urine output) B) Nocturia (voiding at night) C) Bilateral flank pain D) Blood pressure 140/92 E) Severe pain in upper outer quadrant of the abdomen

Ans: A, B Feedback: The symptoms of chronic pyelonephritis often include a history of recurrent episodes of UTI or acute pyelonephritis. Loss of tubular function and the ability to concentrate urine give rise to polyuria and nocturia, and mild proteinuria is common. Severe hypertension often is a contributing factor in the progress of the disease. A BP of 140/92 is not considered "severe" hypertension. Flank and upper outer quadrant pain is usually associated with kidney stones.

A client has been recently undergone diagnostic testing for possible Berger disease. The nurse caring for this client would anticipate the primary clinical manifestations include which of the following? Select all that apply. A) Gross hematuria B) Recent upper respiratory infection C) Elevated ketone levels in the urine D) Fever, chills, and general body aches

Ans: A, B, D Feedback: Early in the disease, many people with the disorder have no obvious symptoms, and the disorder is discovered during screening or examination for another condition. In others, the disorder presents with gross hematuria that is preceded by upper respiratory tract infection, GI tract symptoms, or flulike illness. The hematuria lasts 2 to 6 days. Elevated ketones are usually associated with acidosis, fasting, high-protein diet, or diabetes to name a few.

Which of the following clients should the nurse assess for a type III hypersensitivity immune responses resulting in injury to the vascular system? A client admitted with: Select all that apply. A) Extensive edema associated with acute glomerulonephritis B) Newly diagnosed systemic lupus erythematosus (SLE) C) Facial droop and impaired speech D) Wheezing on inspiration E) Liver failure exhibiting bleeding tendencies

Ans: A,B Feedback: Type III responses create immune complexes that are deposited in the affected tissues, activating an inflammatory response. Autoantibodies are involved with type II responses; cytotoxic cells are involved with type II and type IV responses. Immunoglobulins are released in response to B-cell activation of plasma cells, which does not occur as part of type III reactions.

The nurse can anticipate which of the following interventions to be prescribed for a client with AIDS who is experiencing involuntary weight loss of more than 10% of baseline body weight and frequent diarrhea daily? Select all that apply. A) Enteral or parental nutrition B) Order for megestrol acetate C) Calcium carbonate for mineral replacement D) Benadryl, an antihistamine E) Prenatal vitamins

Ans: A,B Feedback: Wasting syndrome is at least 10% weight loss in the presence of diarrhea, more than two stools a day, or chronic weakness and fever. Treatment for wasting syndrome includes nutritional interventions such as oral supplements or enteral or parenteral nutrition, as well as pharmacologic agents, including appetite stimulants, cannabinoids, and megestrol acetate.

A home health nurse suspects that a child living with his IV drug-abusing mother may have HIV. Which of the following clinical manifestations would confirm this diagnosis? Select all that apply. A) Extremely small for his age to the point of failure to thrive B) CNS abnormalities like seizures, difficulty with walking C) Pneumonia culture reveals a fungal infection of the lungs D) Isolating himself in a back room with the lights dim E) Uncontrolled crying every time the mother approaches

Ans: A,B,C feedback Children may have a different clinical presentation of HIV infection than adults. Failure to thrive, CNS abnormalities, and developmental delays are the most prominent primary manifestations of HIV infection in children. Children born with HIV infection usually weigh less and are shorter than noninfected infants. A major cause of early mortality for HIV-infected children is P. jiroveci pneumonia, which occurs early in children, with the peak age of onset at 3 to 6 months.

Following the diagnosis of nephrotic syndrome, the nurse knows the clinical manifestations occur as a result of a decreased plasma colloidal osmotic pressure. Therefore, the nurse should assess the client for: Select all that apply. A) Moist crackles in both lung fields B) Areas of diminished breath sounds due to pleural effusions C) Liver enlargement D) Kidneys palpable to deep palpation E) Increased circumference in the abdomen related to fluid excess

Ans: A,B,E Feedback: Generalized edema, which is a hallmark of the nephrotic syndrome, results from a decrease in the plasma colloidal osmotic pressure due to the hypoalbuminemia that develops as albumin is lost from the vascular compartment. Initially, the edema presents in dependent parts of the body such as the lower extremities, but becomes more generalized as the disease progresses. Dyspnea due to pulmonary edema, pleural effusions, and diaphragmatic compromise due to ascites (increase fluid in the abdominal cavity) can develop in persons with nephrotic syndrome. Live enlargement is not associated with nephrotic syndrome, but increased synthesis of lipoproteins in the liver secondary to a compensatory increase in albumin production may occur. Palpable kidney mass is associated with cancer.

The clinic nurse suspects the client is having a genetically determined hypersensitivity to common environmental allergens since the client is experiencing which of the following clinical manifestations? Select all that apply. A) Hives B) Runny nose C) Diarrhea D) Topical pustules E) Wheezes

Ans: A,B,E Feedback: Localized, inherited allergic reactions mediated by IgE are known as atopic reactions, such as urticaria (hives). Atopic reactions are immediate (not delayed such as type IV poison ivy rash). Infections are not part of type I hypersensitivity. Autoimmune reactions are in response to antigens, not allergens.

Chlamydiaceae, a rather common sexually transmitted infectious organism, has characteristics of both viruses and bacteria. The infectious form of this organism's life cycle is _______ until it enters the host cell. A) an elementary body B) adhered to cholesterol C) propelled by filaments D) encapsulated hyphae

Ans: A. an elementary body

Although both eukaryotes and prokaryotes are capable of causing infectious diseases in humans, eukaryotes are unique because they have a distinct: A) organized nucleus. B) circular plasmid DNA. C) cytoplasmic membrane. D) variation of shape and size.

Ans: A. organized nucleus

The mechanism by which humans recognize self-cells from non-self (antigens)-cells is called: A) Autoimmunity B) Self-tolerance C) Nonself anergy D) Immunocompatibility

Ans: B Feedback Self-tolerance is the mechanism that humans have for recognizing self-cells from non-self- cells. When tolerance fails, an autoimmune disorder can be the result. Immunocompatibility is how closely the cell surface markers of one human match with those of another human. Anergy is when T cells fail to respond to the presence of an antigen in the body.

A client with a history of emphysema from long-term cigarette smoking has loss of many alveoli. When comparing the type I alveolar cell physiologic function with the primary role of type II alveoli, the nurse would be aware that the type II alveoli are responsible for: A) Facilitation of bronchial circulation B) Production of surfactant C) Gas exchange D) Production of macrophages

Ans: B Feedback: Although macrophages are present in all alveoli, only type II alveoli produce surfactant. They do not participate directly in gas exchange or facilitate bronchial circulation.

Shortly after being diagnosed with HIV, a client has begun highly active antiretroviral therapy (HAART). The client asks, "My doctor tells me that my viral load is going down. What does that mean?" The nurse's best response is: A) "This means that you are in the long-term nonprogressors stage of HIV." B) "Your HAART medications are working to slow the progression of the disease." C) "Your medications are going to decrease your ability to transmit the virus to your sexual partners." D) "You are developing drug resistance and may need to have your medications adjusted."

Ans: B Feedback: Antiviral therapies such as HAART are prescribed to slow the progression of AIDS and improve the overall quality of life and survival time of persons with HIV infection. Extension, not limitation, of the latent period is a goal. Minimizing transmission and preventing seroconversion are not normally achievable goals through drug therapy alone. The final 5% to 15% are long-term nonprogressors, who remain asymptomatic for 10 years or more after seroconversion, with stable CD4+ T-cell counts and low plasma HIV RNA levels.

A client was diagnosed as HIV positive several years ago. Which of the following blood test results reflects the fact that the client now has developed AIDS? A) 350,000 platelets/μL B) CD4+ T-cell count less than 200 cells/μL C) Viral load 500,000 copies/mL D) White blood cell count of 5300 cmm

Ans: B Feedback: CD4+ cell counts are the primary measure that is used for the staging of HIV infection. Viral load, platelets, and leukocytes are also important assessments, but these are not used to stage HIV.

Manifestations of childhood renal disease are varied and may differ from adult-onset renal failure. A school-aged child with chronic kidney disease may exhibit: A) Low IQ level with borderline retardation B) Developmental delays such as uncoordinated gait and minimal fine motor skills C) Inability to control bladder, resulting in incontinence D) Frequent, uncontrolled rolling of the tongue and opening mouth extremely wide

Ans: B Feedback: Childhood chronic kidney disease is manifested by growth and developmental delays and late onset sexual maturity as a result of the uremic effects on endocrine function, bone abnormalities, and development of psychosocial problems. Renal failure is unrelated to the ability of children to have control of urine or bowel elimination. Intelligence is not affected by renal failure, although renal encephalopathy may affect behavior.

1. When lecturing to a group of students about the pathophysiological principles behind heart failure, the instructor explains that cardiac output represents: A) Strength of the right ventricular pump to move blood B) The amount of blood the heart pumps each minute C) The amount of blood pumped out of the heart with each beat D) The volume of blood stretching the heart muscle at the end of diastole

Ans: B feedback Cardiac output, which is the major determinant of cardiac performance, reflects how often the heart beats each minute (heart rate) and how much blood it ejects with each beat (stroke volume). Preload reflects the volume of blood that stretches the ventricle at the end of diastole, just before the onset of systole.

A client who has undergone a liver transplant 7 weeks ago has developed the following assessment data: ALT/AST elevation; jaundice of skin and sclera; weight gain with increase in abdominal circumference; and low-grade fever. The nurse suspects: A) Hyperacute graft rejection B) Acute graft rejection C) Chronic rejection D) Atherosclerosis of arteries of the liver

Ans: B Feedback: Allograph rejection is caused by tissue incompatibility that causes the recipient immune cells to attack and destroy the donor cells. The opposite, graft versus host disease, is characterized by recipient immune cells being destroyed by the donor cells because the recipient cells are recognized by the donor cells as foreign.

A college student has been called into the student health office because she tested positive for HIV on the enzyme-linked immunosorbent assay (ELISA). The student asks, "What is this Western blot assay going to tell you?" The best response by the health care provider is: A) "We always want two positive test results before we give you medicine." B) "The Western blot is a more sensitive assay that looks for the presence of antibodies to specific viral antigens." C) "This assay will actually look at all the individual cells in your blood and count how many HIV cells you have, so we can treat you with the proper medication." D) "If you are afraid of another blood test, we can do a rapid oral test to see if we get the same results."

Ans: B Feedback: The Western blot is a more sensitive assay than the EIA that looks for the presence of antibodies to specific viral antigens. In the case of a false-positive EIA result, the Western blot test can identify the person as uninfected. Technologic advances have led to new forms of testing, such as the oral test, home testing kits, and the new rapid blood test. Oral fluids contain antibodies to HIV. In the late 1990s, the FDA approved the OraSure test. The OraSure uses a cotton swab, which is inserted into the mouth for 2 minutes, placed in a transport container with preservative, and then sent to a laboratory for EIA and Western blot testing.

The nurse caring for a population of HIV clients needs to be assessing for which of the following diseases that amounts to the leading cause of death for people with HIV? A) Leukemia B) Tuberculosis C) Pneumonia D) Toxoplasmosis

Ans: B feedback Although HIV-positive people can develop all types of cancer, have viral or bacterial pneumonia, and be infected with the parasite that causes toxoplasmosis, tuberculosis is the disease that leads to death most often. Tuberculosis (TB) is the leading cause of death for people with HIV infection worldwide and is often the first manifestation of HIV infection. In 2011, 23% of those with TB tested positive for HIV.

A client with a diagnosis of aplastic anemia has undergone allogenic bone marrow transplantation. Which of the following signs and symptoms would most clearly suggest the existence of graft versus host disease (GVHD)? A) Shortness of breath, audible crackles, and decreasing PaO2 B) Presence of a pruritic rash that has begun to slough off C) Development of metabolic acidosis D) Diaphoresis, fever, and anxiety

Ans: B feedback In cases of GVHD, there is development of a pruritic, maculopapular rash that begins on the palms and soles and frequently extends over the entire body, with subsequent desquamation. Respiratory complications are atypical, and neither metabolic acidosis nor diaphoresis and fever are common.

A client with heart failure asks, "Why am I taking a 'water pill' when it's my heart that is having a problem?" While educating the client about the Frank-Starling mechanism, which of the following explanations is most appropriate to share? A) "You must be drinking way too many liquids. Your kidneys cannot filter all that you are drinking during the day." B) "Since your heart is not pumping efficiently, the kidneys are getting less blood flow; therefore, the kidneys are holding on to sodium and water." C) "Your heart muscle is overstretched, so it's not able to pump all the blood out. The prescribed 'water pills' help by decreasing your weight." D) "Since your heart function is impaired, the lungs are not able to oxygenate the blood and your kidneys are wearing out."

Ans: B feedback In heart failure with a reduced ejection fraction, a decrease in cardiac output and renal blood flow leads to increased sodium and water retention by the kidney with a resultant increase in vascular volume and venous return to the heart and an increase in ventricular end-diastolic volume. Drinking water may increase volume but is not the physiological reason for retention of fluid. Diuretics do decrease weight as a result of diuresis, but weight loss is not the purpose for giving diuretics. The lungs are not the primary cause of heart failure.

Which of the following statements from a group of young adults demonstrates the need for further teaching related to HIV and prevention of the spread? A) "A woman can still get pregnant if she is HIV positive." B) "Having oral sex is one way I can prevent passing on HIV to my partner." C) "Good perineal care following sex will not have an effect on the transmission of HIV." D) "My HIV medications do not prevent me from passing the virus on to my partners."

Ans: B feedback Sexual contact is the most frequent mode of HIV transmission. There is a risk of transmitting HIV when semen or vaginal fluids come in contact with a part of the body that lets them enter the bloodstream. This can include the vaginal mucosa, anal mucosa, and wounds or sores on the skin. Condoms are highly effective in preventing the transmission of HIV. Unprotected sex between men is still the main mode of transmission. During the window period, a person's HIV antibody test result will be negative, but he or she can still transmit the virus.

Although growth rate is variable among types of bacteria, the growth of bacteria is dependent on: A) biofilm communication. B) availability of nutrients. C) an intact protein capsid. D) individual cell motility.

Ans: B availability of nutrients

A patient with a history of heart failure has been referred for an echocardiogram. Results of this diagnostic test reveal the following findings: heart rate 80 beats per minute; end- diastolic volume 120 mL; end-systolic volume 60 mL. What is this patient's ejection fraction? A) 200 mL B) 50% C) .80 D) 180 mL

Ans: B) 50%

In the days following a tooth cleaning and root canal, a patient has developed an infection of the thin, three-layered membrane that lines the heart and covers the valves. What is this patient's most likely diagnosis? A) Pericarditis B) Endocarditis C) Myocarditis D) Vasculitis

Ans: B) Endocarditis

Which of the following factors is the primary governor of the local control of blood flow? A) Action potential B) The nutritional needs of the tissue involved C) Cardiac contractility and preload D) Feedback from arterial baroreceptors and chemoreceptors

Ans: B) The nutritional needs of the tissue involved

Atherosclerotic peripheral vascular disease is symptomatic with at least 50% occlusion. The primary peripheral symptom, due to ischemia, is: A) edema. B) calf pain. C) varicosities. D) strong pulse.

Ans: B) calf pain.

Treponema pallidum, the cause of syphilis, is a spirochete bacterium that is spread from human to human by: A) tick or lice vector bites. B) direct physical contact. C) exposure to infected urine. D) inhaling airborne particles.

Ans: B) direct physical contact.

In the arterial-venous circulatory system, pressure is inversely related to: A) velocity. B) volume. C) tension. D) viscosity.

Ans: B) volume.

Which of the following lab values correlate to the fact that the client has developed a lytic drug reaction and is having an antibody response that lyses the drug-coated cell? Select all that apply. A) CD4 T-cell count of 500 cells B) Low hemoglobin of 9.1 C) WBC level of 5000 D) Low platelet level of 10,000 E) Elevated liver enzyme levels

Ans: B, D Feedback: Lytic drug reactions can produce transient anemia (low RBCs and hemoglobin), leukopenia, or thrombocytopenia (low platelet levels), which often are corrected by the removal of the offending drug. A WBC of 5000 is normal. Changes in CD4 T cell counts relate to HIV infection. Liver enzymes do not rise because of a drug reaction.

Client and family education regarding peritoneal dialysis should include assessing the client for: A) Bleeding around the arteriovenous fistula or an external arteriovenous shunt B) Signs and symptoms of hypoglycemia such as weakness, irritability, and shakiness C) Dehydration that may appear as dry mucous membranes or poor skin turgor D) Muscle cramps associated with hypoparathyroidism

Ans: C Feedback: Potential problems with peritoneal dialysis include infection, catheter malfunction, dehydration caused by excessive fluid removal, hyperglycemia, and hernia. The most serious complication is infection, which can occur at the catheter exit site, in the subcutaneous tunnel, or in the peritoneal cavity. In peritoneal dialysis, a sterile dialyzing solution is instilled through a catheter over a period of approximately 10 minutes. Then the solution is allowed to remain in the peritoneal cavity for a prescribed amount of time. Shunts, fistulas, and artificial dialyzers are associated with hemodialysis, which is usually performed three times weekly.

A client with a diagnosis of cirrhosis has experienced an acute rejection of a donor liver. Which of the following cells is responsible for the rejection of the client's transplanted organ? A) Natural killer cells B) Mast cells C) T cells D) Neutrophils

Ans: C Feedback: Although different patterns of rejection exist, a general commonality is that T cells play a central role in the process of rejection. Mast cells and leukocytes do not participate directly in the physiology of rejection

Following several weeks of increasing fatigue and a subsequent diagnostic work-up, a patient has been diagnosed with mitral valve regurgitation. Failure of this heart valve would have which of the following consequences? A) Backup of blood from the right atrium into the superior vena cava B) Backflow from the right ventricle to the right atrium during systole C) Inhibition of the SA node's normal action potential D) Backflow from the left ventricle to left atrium

Ans: D) Backflow from the left ventricle to left atrium

While working in the newborn nursery, a nurse is assessing a new admission. The nurse notes the infant has an increased distance between his eyes, a very small jaw, and a split uvula. Thinking this infant might have DiGeorge syndrome, the nurse should be assessing this infant for which of the following electrolyte imbalances? A) Frequent ventricular beats on ECG due to hyperkalemia B) Loss of consciousness due to hyponatremia C) Tetany due to hypocalcemia D) Decreased reflexes due to hypermagnesemia

Ans: C Feedback: Infants born with this defect have partial or complete failure in development of the thymus and parathyroid glands and have congenital defects of the head, neck, and/or heart. The facial disorders can include hypertelorism (increased distance between the eyes); micrognathia (abnormally small jaw); low-set, posteriorly angulated ears; split uvula; and high-arched palate. Urinary tract abnormalities also are common. The most frequent presenting sign is hypocalcemia and tetany that develop within the first 24 hours of life. It is caused by the absence of the parathyroid gland and is resistant to standard therapy.

Contact with poison ivy has resulted in intense pruritus, erythema, and weeping on a client's forearm. Which of the following processes resulted in the client's signs and symptoms? A) IgE-mediated mast cell degranulation B) Formation of antigen-antibody complexes C) Cytokine release by sensitized T cells D) Formation of antibodies against cell surface antigens

Ans: C Feedback: The characteristic reaction to poison ivy is an example of contact dermatitis, a type IV hypersensitivity reaction. As such, the reaction is caused by sensitized T cells and the release of cytokines. IgE-mediated mast cell degranulation causes type I reactions, whereas antigen-antibody complexes are associated with type III reactions. Antibody formation against cell surface antigens are associated with type II reactions.

A 70-year-old female client has had her mobility and independence significantly reduced by rheumatoid arthritis. Which of the following processes likely contributed to the development of her health problem? A) Delayed-type hypersensitivity (DTH) reaction B) Proliferation of cytotoxic T cells C) Failure of normal self-tolerance D) Deletion of autoreactive B cells

Ans: C Feedback: Ultimately, all autoimmune diseases are attributable to a disruption in self-tolerance that results in damage to body tissues by the immune system. Autoimmune diseases are not known to result from DTH reactions or the proliferation of cytotoxic T cells. Deletion of autoreactive B cells is a normal process that prevents autoimmune reactions in healthy individuals.

A child has experienced a "bee-sting" while at the park. The health care provider is walking by and notices the child has swelling around the eyes, lips, and face in general. What priority assessment should the nurse make at this time? A) Palpate for carotid pulses in the neck. B) Assess skin on the truck and back for development of hives. C) Assess and establish an open airway. D) Try to listen to breath sounds by placing

Ans: C feedback The initial management of anaphylaxis focuses on the establishment of a stable airway and intravenous access and the administration of epinephrine. Epinephrine produces relaxation of bronchial smooth muscle and inhibits the immediate life- threatening cardiovascular effects of anaphylaxis. All the other assessments (pulse rate, skin, and breath sounds) can be done after the airway is assessed and managed.

Because dermatophytes are capable of growing _________, the infection is mainly found on cutaneous surfaces of the body. A) a powdery colony B) in moist skin folds C) on cooler tissue D) branching filaments

Ans: C on cooler tissue

Harmful effects on cardiac action potential are most likely to result from a deficit of which of the following electrolytes? A) Magnesium (Mg2+) B )Chloride (Cl-) C) Potassium (K+) D) Hydrogen carbonate (HCO3-)

Ans: C) Potassium (K+)

A male patient with a history of angina has presented to the emergency department with uncharacteristic chest pain and his subsequent ECG reveals T-wave elevation. This finding suggests an abnormality with which of the following aspects of the cardiac cycle? A) Atrial depolarization B) Ventricular depolarization C) Ventricular repolarization D) Depolarization of the AV node, bundle branches, and Purkinje system

Ans: C) Ventricular repolarization

A large increase in heart rate can cause: A) increased blood viscosity. B) loss of action potential. C) decreased stroke volume. D) reduced cardiac contractility.

Ans: C) decreased stroke volume.

The tissue factor that contributes to humoral control of blood flow by causing vasoconstriction is: A) histamine. B) bradykinin. C) serotonin. D) nitric oxide.

Ans: C) serotonin.

The parasympathetic nervous system causes a slowing of the heart rate by increasing: A) norepinephrine. B) vessel constriction. C) vagus nerve activity. D) smooth muscle tone.

Ans: C) vagus nerve activity.

A client has been admitted to the hospital for the treatment of HIV infection, which has recently progressed to overt AIDS. Which of the following nursing actions should the nurse prioritize when providing care for this client? A) Frequent neurologic vital signs and thorough skin care B) Hemodynamic monitoring and physical therapy C) Careful monitoring of fluid balance and neurologic status D) Astute infection control and respiratory assessments

Ans: D Feedback: Although all of the cited assessments and interventions may be of some value, infection control and the early identification of potential respiratory infections are paramount in the care of clients with AIDS.

HIV-positive persons who display manifestations of laboratory category 3 or clinical category C are considered to have: A) Zero viral load B) Seroconversion C) Complete remission D) AIDS-defining illnesses

Ans: D Laboratory category 3 and clinical category C are the most serious and indicate the presence of AIDS-defining illnesses. Seroconversion is clinical category A; zero viral load and remission could also be category A.

A client has developed pericarditis following an episode of acute glomerulonephritis, developments that may be attributable to the presence of similar epitopes on group A beta- hemolytic streptococci and the antigens in the client's heart tissue. Which of the following has most likely accounted for this client's autoimmune response? A) Breakdown of T-cell anergy B) Release of sequestered antigens C) Superantigens D) Molecular mimicry

Ans: D feedback Molecular mimicry is the phenomenon that exists when a microbe shares an immunologic epitope with the host, precipitating an autoimmune response. This miscategorization by cells of the immune system does not result from the release of sequestered antigens, failure of T-cell anergy, or the presence of superantigens.

In an ICU setting, one assessment that would lead the nurse to suspect shock has resulted in decrease blood flow to vital organs is: A) Warm legs with peripheral vasodilation B) Urine output less than 20 mL/hour C) Blood pressure staying in the 98/72 range for the past hour D) Sleepiness and difficulty to arouse without using painful stimuli

B Feedback: Continuous measurement of urine output is essential for assessing the circulatory status of a person in shock. Oliguria of 20 mL/hour or less indicates inadequate renal perfusion. Continuous measurement of urine output is essential for assessing the circulatory and volume status of the person in shock and monitoring the response to fluid replacement. As the shock progresses and blood flow to the brain decreases, restlessness is replaced by apathy and stupor. Sympathetic stimulation also leads to intense vasoconstriction of the skin vessels and activation of the sweat glands. As a result, the skin is cool and moist. There is an increase in heart rate, cool and clammy skin, a decrease in arterial blood pressure, and a decrease in urine output.

A teenaged cystic fibrosis client presents to the clinic. The health care provider (HCP) knows that cystic fibrosis (CF) causes severe chronic respiratory disease in children. In addition, the HCP should also focus his or her assessment on which of the other body systems affected by CF? A) Renal B) Pancreatic C) Cardiac D) Central nervous system

B Feedback: Cystic fibrosis (CF) is manifested by pancreatic exocrine deficiency and elevation of sodium chloride in the sweat. Cystic fibrosis (CF) is an inherited disorder involving fluid secretion by the exocrine glands in the epithelial lining of the respiratory, gastrointestinal, and reproductive tracts. Excessive loss of sodium in the sweat predisposes young children to salt depletion episodes. Respiratory manifestations are caused by an accumulation of viscid mucus in the bronchi, impaired mucociliary clearance, lung infections, bronchiectasis, and dilatation. The renal, cardiac, and CNS are usually not involved with CF manifestation.

An elderly client who experiences chronic pain takes opioid analgesics on a regular basis, a practice that has resulted in frequent constipation and occasional bowel obstructions. Which of the following problems may directly result from these gastrointestinal disorders? A) Urinary tract infections B) Overflow urinary incontinence C) Bladder cancer D) Neurogenic bladder

B Feedback: Fecal impaction occurs when a large bolus of stool forms in the rectum, which can push against the urethra causing obstruction that results in overflow incontinence. This does not constitute a risk factor for bladder cancer or neurogenic bladder, and although a urinary tract infection (UTI) may result, this is an indirect consequence of the bowel obstruction.

When evaluating the pulmonary function test results for a COPD client, which one correlates to the mismatch of ventilation and perfusion associated with this diagnosis? A) Forced vital capacity (FVC) is elevated. B) Forced expiratory volume (FEV) is decreased. C) Total lung capacity (TLC) is decreased. D) Marked decrease in residual volume (RV).

B Feedback: In clients with chronic lung disease, the FVC is decreased, the FEV1.0 is decreased, and the ratio of FEV1.0 to FVC is decreased. Lung volume measurements reveal a marked increase in RV, an increase in TLC, and elevation of the RV-to-TLC ratio.

A client with significant burns on his lower body has developed sepsis on the 3rd day following his accident. Which of the following manifestations would the nurse anticipate for an ischemic acute tubular necrosis rather than prerenal failure? The client: A) Exhibits pulmonary and peripheral edema B) GFR does not increase after restoration of renal blood flow C) Undergoes emergent hemodialysis that does not result in decreased BUN and creatinine D) Exhibits oliguria and frank hematuria

B Feedback: In contrast to prerenal failure, the glomerular filtration rate (GFR) does not improve with the restoration of renal blood flow in acute renal failure caused by ischemic acute tubular necrosis. Edema, oliguria, and hematuria are not diagnostic of acute tubular necrosis (ATN), and hemodialysis does not normally fail to achieve a reduction in blood urea nitrogen (BUN) and creatinine.

Which of the following client clinical manifestations most clearly suggests a need for diagnostic testing to rule out renal cell carcinoma? A) Urinary urgency B) Hematuria C) Oliguria D) Cloudy urine

B Feedback: Presenting features of renal cancer include hematuria, flank pain, and presence of a palpable flank mass. Gross or microscopic hematuria, which occurs in more than 50% of cases, is an important clinical clue. Urgency, oliguria, and cloudy urine are not as closely associated with renal carcinoma.

Reduced glomerular filtration rate (GFR), with a serum creatinine level that remains in the normal range, is associated with aging because elderly persons tend to have reduced: A) Calcium intake B) Muscle mass C) Drug tolerance D) Renal perfusion

B Feedback: Serum creatinine level is directly related to muscle metabolism. Because muscle mass is reduced in elderly persons, the creatinine level does not increase as readily with a lower GFR. Drug tolerance and renal perfusion can affect the GFR, but the age-related normal creatinine level can also be present. Calcium intake is unrelated to creatinine levels or GFR.

Which one of the following blood tests reflects the glomerular filtration rate (GFR) and is used to estimate renal function? A) Blood protein B) Serum creatinine C) Serum ammonia D) Blood urea nitrogen

B Feedback: Serum creatinine level is used to estimate functional capacity of the kidneys. Increased creatinine level indicates decreased GFR and renal function. Blood urea nitrogen (BUN) levels are influenced by hydration status, protein intake, and bleeding, in addition to renal function. Serum ammonia is a metabolic by-product of urea and can be influenced by multiple factors unrelated to kidney function.

COPD clients live with persistently elevated PCO2 levels. Therefore, which assessment finding would likely initiate a stimulus for ventilation in this client population? A) PCO2 level of 65 mm Hg B) PO2 level of 50 mm Hg C) Arterial blood gas pH of 7.35D) D) Pulse oximeter reading of 96% saturation

B Feedback: The central chemoreceptors are extremely sensitive to short-term changes in blood PCO2 levels. An increase in the PCO2 of the blood produces an increase in ventilation that reaches its peak within a minute or so and then declines if the PCO2 level remains elevated. Thus, persons with chronically elevated blood PCO2 levels no longer respond to this stimulus for increased ventilation but rely on the stimulus provided by a decrease in arterial PO2 levels that is sensed by the peripheral chemoreceptors.

A college student is training for a marathon in the mountains. One day, she experiences a sharp pain and suddenly becomes short of breath. At the emergency room, chest x-ray reveals a spontaneous pneumothorax. The client asks the nurse to explain why this happened. The nurse states, "For unknown reasons, you lose intrapleural negative pressure. A) You must have experienced a forced expiration against a closed glottis to cause the lung to deflate." B) This means your lungs collapsed and expelled its air when you lose negative pressure." C) You must have coughed too forcibly and your air sacs burst." D) You must have a genetic anomaly causing weakened alveolar sacs to rupture."

B Feedback: The intrapleural pressure is always negative in relation to alveolar pressure in the normally inflated lung: approximately 4 mm Hg between breaths when the glottis is open and the alveolar spaces are open to the atmosphere. Although the intrapleural pressure of the inflated lung is always negative in relation to alveolar pressure, it may become positive in relation to atmospheric pressure. Although the intrapleural pressure of the inflated lung is always negative in relation to alveolar pressure, it may become positive in relation to atmospheric pressure (e.g., as during forced expiration and coughing). A spontaneous pneumothorax is a collapsed lung with loss of negative pressures.

A client is beginning to recover from acute tubular necrosis. The nurse would likely be assessing which of the following manifestations of the recovery phase of ATN? A) Edema B) Diuresis C) Proteinuria D) Hypokalemia

B Feedback: The recovery phase is first noticed as increased/excessive output (diuresis) of dilute urine and a fall in serum creatinine, indicating that the nephrons have recovered to the point at which urine excretion is possible. Potassium will remain elevated or continue to rise, since the diuresis occurs before renal function fully returns to normal. Edema/fluid retention is characteristic of the maintenance phase. Proteinuria is characteristic of glomerular disease and/or chronic kidney disease.

When the glomerular transport maximum for a substance such as blood glucose is exceeded and its renal threshold has been reached, the substance will: A) Reabsorb quickly B) Spill into the urine C) Countertransport sodium D) Attach to protein carriers

B Feedback: When the substance (such as blood glucose) exceeds the number of carrier proteins available for transport, the transport maximum has been exceeded, the renal threshold is reached, and the substance will spill (not reabsorb) into the urine. Sodium cotransport helps to move the substance back into the tubule.

A client has provided a routine urine sample during a scheduled visit to his primary care provider. Which of the following results is an expected finding in a healthy individual? A) Low to moderate amount of glucose in the urine B) Urine specific gravity of 1.020 C) Presence of moderate amounts of albumin with absence of other proteins D) Presence of urinary casts

B feedback Normal urine specific gravity ranges from 1.010 to 1.025. Glucose and casts are normally absent, and albumin is normally present in only scant amounts.

Which of the following physiologic processes is performed by the kidneys and contributes to increased blood pressure? A) Catalysis of the conversion of angiotensin I to angiotensin II B) Production and release of renin C) Secretion of aldosterone D) Conversion of aldosterone to angiotensin

B feedback Renin, an enzyme that is synthesized and stored in the juxtaglomerular cells of the kidney, enzymatically converts angiotensinogen to angiotensin I. Angiotensin I is converted to angiotensin II, a potent vasoconstrictor, in the lungs. Aldosterone is secreted by the adrenal glands and does not convert to angiotensin.

Prior to undergoing diagnostic testing with contrast, it is recommended that older adult clients have their creatinine level checked. The rationale for this is to ensure the client: A) Is not allergic to shell fish or iodine B) Will not undergo an acute kidney injury by decreasing renal blood flow C) Does not have a kidney stone obstructing the urethra D) Is in good enough health to withstand a walking on a treadmill

B feedback: Some drugs, such as diuretics, high molecular weight radiocontrast media, the immunosuppressive drugs cyclosporine and tacrolimus, and the nonsteroidal anti- inflammatory drugs (NSAIDs), can cause acute kidney injury by decreasing renal blood flow. Checking creatinine levels do not predict the client's allergies, a kidney stone, or tolerance for stress testing.

A client with a history of chronic pyelonephritis has been admitted several times with recurrent bacterial infection of the urinary tract. The nurse should anticipate educating this client with regard to which common treatment regimen? A) Increase intake of cranberry juice to 2 L/day. B) Continue taking antibiotics for full 10 to 14 days even if symptoms of infection disappear. C) Force micturition every 2 hours while awake. D) Take prescribed diuretics early in the day to avoid having to get up during the night.

B feedback: Chronic pyelonephritis involves a recurrent or persistent bacterial infection superimposed on urinary tract obstruction, urine reflux, or both. Chronic obstructive pyelonephritis can be bilateral, caused by conditions that obstruct bladder outflow; or unilateral, such as occurs with ureteral obstruction. Cranberry juice, forced micturition, and diuretics are not standard treatments for chronic pyelonephritis.

A public health nurse is conducting a health promotion class for a group of older adults. Which of the participants' following statements demonstrates an accurate understanding of the risk factors for bladder cancer? A) "I suppose I should listen to my doctor and drink more cranberry juice." B) "More than ever, I guess it would worthwhile for me to quit smoking." C) "I can see that preventing bladder cancer is one more benefit of a healthy diet." D) "I think I should be okay because there's no history of bladder cancer in my family that I'm aware of."

B Feedback: Smoking is implicated in 30% to 50% of all bladder cancers among males who are current or past smokers. Cranberry juice may be of benefit in the prevention of UTIs, not cancer, and neither poor diet nor family history is as significant as cigarette smoking in the etiology of bladder cancer.

Which of the following assessment findings would lead the nurse to suspect the client has nephrotic syndrome? A) Hematuria and anemia B) Proteinuria and generalized edema C) Renal colic and increased serum sodium D) Increased creatinine with normal blood urea nitrogen

B Feedback: The nephrotic syndrome is characterized by massive proteinuria and lipiduria, along with an associated hypoalbuminemia, generalized edema, and hyperlipidemia.

Which of the following medications may be responsible for a client developing increased uric acid levels by decreasing ECF volume? A) Vitamin C B) Thiazide diuretics C) Penicillin antibiotics D) Maalox products

B feedback Because of its effect on uric acid secretion, aspirin is not recommended for treatment of gouty arthritis. Thiazide and loop diuretics also can cause hyperuricemia and gouty arthritis, presumably through a decrease in ECF volume and enhanced uric acid reabsorption

The nurse is scheduled to teach a client experiencing urinary incontinence about Kegel exercises. Which of the following descriptors should the nurse include in this education? A) "Drink at least two glasses of water and then try to hold it for at least 3 hours before going to the bathroom." B) "Contract and relax the pelvic floor muscles at least 10 times every hour while awake." C) "After you have emptied your bladder, continue sitting on the commode and try to forcefully expel more urine." D) "Try to start and stop urination while sitting in a bathtub full of warm soapy water."

B feedback Exercises for the pelvic muscles or Kegel exercises involve repetitive contraction and relaxation of the pelvic floor muscles and are an essential component of client-dependent behavioral interventions. None of the other distractors are examples of Kegel exercises.

For which of the following types of shock might intravenous antibiotic therapy be indicated? A) Obstructive shock B) Distributive shock C) Cardiogenic shock D) Hypovolemic shock

B feedback Septic shock is a subtype of distributive shock. The treatment of sepsis and septic shock focuses on control of the causative agent and support of the circulation and the failing organ systems. The administration of antibiotics that are specific for the infectious agent is essential. Swift and aggressive fluid administration is needed to compensate for third spacing, though which type of fluid is optimal remains controversial. Equally, aggressive use of vasopressor agents, such as norepinephrine or epinephrine, is needed to counteract the vasodilation caused by inflammatory mediators.

A female teenager has experienced three uncomplicated urinary tract infections in the last 3 months. Knowing the anatomical location of the urethra, the nurse should educate this teenager about: A) Proper handwashing to decrease amount of Pseudomonas growing on the hands B) Wiping from front to back to prevent Escherichia coli contamination of the urethra C) Wearing gloves when wiping perineum after defecation to prevent Staphylococcus aureus infection D) Washing hands prior to inserting a tampon to minimize the risk of group B Streptococcus

B feedback Most commonly, urinary tract infections (UTIs) are caused by Escherichia coli that enter through the urethra. Escherichia coli are abundant in fecal matter. Other uropathic pathogens include Staphylococcus saprophyticus in uncomplicated UTIs and both non-E. coli gram-negative rods (Proteus mirabilis, Klebsiella pneumoniae, Pseudomonas) and gram-positive cocci (Staphylococcus aureus, group B Streptococcus) in complicated UTIs.

A patient's recent history of emphysema has resulted in the functional loss of many alveoli. Which of the following physiologic functions is the primary role of type II alveoli? A) Facilitation of bronchial circulation B) Production of surfactant C) Gas exchange D) Production of macrophages

B) Production of surfactant

An athlete has become dehydrated during a long race in hot weather. Which of the following physiologic processes will occur in an attempt to protect the athlete's extracellular fluid volume? A) Dilation of the afferent and efferent arterioles B) Release of ADH from the posterior pituitary C) Increased water reabsorption in the ascending limb of the loop of Henle D) Increased water reabsorption in the distal convoluted tubule

B feedback: ADH assists in the maintenance of the extracellular fluid volume by controlling the permeability of the medullary collecting tubules. Osmoreceptors in the hypothalamus sense an increase in osmolality of extracellular fluids and stimulate the release of ADH from the posterior pituitary gland. In exerting its effect, ADH, also known as vasopressin, binds to receptors on the basolateral side of the tubular cells. Binding of ADH to the vasopressin receptors causes water channels, known as aquaporin-2 channels, to move into the luminal side of the tubular cell membrane, producing a marked increase in water permeability. The ascending limb of the loop of Henle and the distal convoluted tubule are largely impermeable to water, and arteriole dilation does not directly increase the amount of water reabsorbed from glomerular filtrate.

Which of the following statements about the use of angiotensin-converting enzyme inhibitor medications and autosomal recessive polycystic kidney disease (ARPKD) is accurate? A) The use of ACE inhibitors will increase the vasopressin levels. B) ACE inhibitors may interrupt the renin- angiotensin-aldosterone system to reduce renal vasoconstriction. C) The ACE inhibitors have been shown to shrink the size of the cysts inside the kidneys. D) ACE inhibitors should be used strictly in those clients who also have an underlying cardiac history.

B feedback: In addition to increasing water intake to decrease vasopressin levels, the angiotensin- converting enzyme (ACE) inhibitors or angiotensin II receptor blockers (ARBs) may be used to interrupt the renin-angiotensin- aldosterone system as a means of reducing intraglomerular pressure and renal vasoconstriction. Although not approved by the Food and Drug Administration (FDA), there has been recent interest in the use of vasopressin receptor antagonists (vaptans) to decrease cyst development.

Which of the following events would suggest that an individual's physiologic response to an obstruction has progressed beyond the compensatory stage and is now in the decompensatory stage? A) The bladder muscle hypertrophies. B) The detrusor loses its power of contraction. C) The ability to suppress urination is diminished D) The individual experiences urgency.

B feedback: The compensatory stage of the response to a bladder obstruction involves hypertrophy of the bladder wall, urgency, and difficulty suppressing the urge to urinate. If these compensatory measures are ineffective, decompensation occurs. The period of detrusor muscle contraction becomes too short to expel the urine completely, and residual urine remains in the bladder. At this point, the symptoms of obstruction— frequency of urination, hesitancy, need to strain to initiate urination, a weak and small stream, and termination of the stream before the bladder is completely emptied—become pronounced.

Which of the following individuals is experiencing a health problem that is the result of a parasite? A) A college student who contracted Chlamydia trachomatis during an unprotected sexual encounter B) A man who acquired malaria while on a tropical vacation C) A hospital patient who has developed postoperative pneumonia D) A woman who developed hepatitis A from eating at an unhygienic restaurant

B) A man who acquired malaria while on a tropical vacation

Which of the following individuals is suffering the effects of acute coronary syndrome (ACS)? A) A patient whose most recent ECG indicates that silent myocardial ischemia has occurred B) A patient who occasionally experiences persistent and severe chest pain when at rest C) A patient who sometimes experiences chest pain when climbing stairs D) A patient who has recently been diagnosed with variant (vasospastic) angina

B) A patient who occasionally experiences persistent and severe chest pain when at rest

A patient with a diagnosis of chronic renal failure secondary to diabetes has seen a gradual increase in her blood pressure over the past several months, culminating in a diagnosis of secondary hypertension. Which of the following has most likely resulted in the patient's increased blood pressure? A) Increased levels of adrenocortical hormones B) Activation of the renin-angiotensin-aldosterone mechanism C) Increased sympathetic stimulation by the autonomic nervous system (ANS) D) Coarctation of the patient's aorta

B) Activation of the renin-angiotensin-aldosterone mechanism

Which of the following forms accounts for the greatest percentage of carbon dioxide transport? A) Free CO2 B) Bicarbonate C) Dissolved CO2 D) Attached to hemoglobin

B) Bicarbonate

The entrance of a microbe into an individual's vascular space has initiated opsonization. Which of the following processes is involved in opsonization? A) Stimulation of B cells by helper T cells B) Coating of a microbe to aid phagocyte recognition C) Release of proteins that stimulate cell production by the bone marrow D) Lysis of intracellular microbes by cytotoxic T cells

B) Coating of a microbe to aid phagocyte recognition

Patients with ischemic coronary vessel disease and acute coronary syndrome (ACS) are classified as low or high risk for acute myocardial infarction based on characteristics that include significant: A) heart murmurs. B) ECG changes. C) pulmonary disease. D) pericardial effusion.

B) ECG changes.

A 1-day-old infant was exposed to an infectious microorganism prior to discharge home from the hospital, but was able to effect a sufficient immune response in the hours and days following exposure. This immune response may have been due to the presence of which of the following immunoglobulins from the infant's mother? A) IgA B) IgG C) IgM D) IgD

B) IgG

The first circulating immunoglobulin to appear in response to a new antigen is: A) IgG. B) IgM. C) IgA. D) IgD.

B) IgM.

Which of the following respiratory pressures is always negative in relation to alveolar pressure? A) Airway pressure B) Intrapleural pressure C) Intrathoracic pressure D) Intrapulmonary pressure

B) Intrapleural pressure

A child's thymus gland is fully formed and proportionately larger than an adult's. Which of the following processes that contribute to immunity takes place in the thymus gland? A) Differentiation of B cells B) Production of natural killer (NK) cells C) Proliferation of T cells D) Filtration of antigens from the blood

B) Production of natural killer (NK) cells

Which of the following characteristics of the lungs of infants and small children creates an increased risk of respiratory disorders? A) Type II alveoli in children may overproduce surfactant. B) Smaller airways create a susceptibility to changes in airway resistance and airflow. C) The pneumotaxic center in the pons is underdeveloped until 8 years of age. D) There are fewer chemoreceptors in the young medulla.

B) Smaller airways create a susceptibility to changes in airway resistance and airflow. Feedback: Because the resistance to airflow is inversely related to the fourth power of the radius (resistance = 1/radius), relatively small amounts of mucus secretion, edema, or airway constriction can produce marked changes in airway resistance and airflow. Surfactant production is low early in life, and the respiratory center and chemoreceptors are present and functional in infants and children.

International travel has contributed to increased prevalence and incidence of nonindigenous diseases by increasing which of the following? A) Portals of entry B) Sources of infection C) Virulence D) Disease course

B) Sources of infection

22-11. Which of the following medications helps treat the inflammatory reaction of asthma clients diagnosed with late-phase asthma response? A) Anticholinergic agents B) Systemic corticosteroids C) Long-acting β2-agonists D) Phosphodiesterase inhibitors

B) Systemic corticosteroids Feedback: A short course of systemic corticosteroids, administered orally or parenterally, may be used for treating the inflammatory reaction associated with the late-phase response. The anticholinergic agents block cholinergic receptors and reduce intrinsic vagal tone that causes bronchoconstriction. The long-acting β2 agonists, available for administration by the inhaled or oral routes, act by relaxing bronchial smooth muscle. Theophylline, a phosphodiesterase inhibitor, is a bronchodilator that acts by relaxing bronchial smooth muscle.

Respiratory movement of air that does not participate in alveolar gas exchange is known as alveolar dead space. Dead space increases when alveoli: A) carbon dioxide level is high. B) air supply exceeds blood flow. C) contain pulmonary edema fluid. D) collapse onto the capillary bed.

B) air supply exceeds blood flow.

In addition to being the site of gas exchange, the lungs also: A) activate vasoactive substances. B) contain cells that produce heparin. C) empty extra blood volume into the left heart. D) use water vapor pressure to inflate alveoli.

B) contain cells that produce heparin.

Above the glottis that opens and closes for speech, the epiglottis functions to _______ the larynx during swallowing. A) open B) cover C) collapse D) constrict

B) cover

Once T helper cells are activated, they secrete ____________ that activate and regulate nearly all of the other cells of the immune system. A) complement proteins B) cytokines C) leukotrienes D) bradykinins

B) cytokines

The effector cells of the immune system have the primary function of: A) activating phagocytic cells. B) eliminating the antigens. C) processing antigen into epitopes. D) controlling the immune response.

B) eliminating the antigens.

The oxyhemoglobin buffer system changes with the metabolic needs of the tissues. When the change causes a shift to the right in the dissociation curve, it is often the result of: A) decreased CO2 levels. B) increase tissue metabolism. C) decreased body temperature. D) increase red blood cell volume.

B) increase tissue metabolism.

Adaptive immune responses, also called acquired or specific immunity, are composed of _____________ and their products. A) granulocytes B) lymphocytes C) epithelial cells D) Toll-like receptors

B) lymphocytes

Innate immunity, also called natural or native immunity, consists of mechanisms that respond specifically to: A) self-cells. B) microbes. C) antibodies. D) inflammation.

B) microbes.

The function of the mucociliary blanket that lines the conducting airways is to: A) dehumidify inhaled air. B) remove foreign materials. C) warm the moving gases. D) spread antibacterial enzymes.

B) remove foreign materials

The most important complication of atherosclerosis is _________, which may cause occlusion of small heart vessels. A) ulceration B) thrombosis C) fatty streaks D) fibrous plaque

B) thrombosis

A diabetic client with a history of hypertension may receive a prescription for which medication to provide a renal protective effect by reducing intraglomerular pressure? Select all that apply. A) Loop diuretics B) ACE inhibitors C) Angiotensin receptor blockers D) Calcium channel blockers E) A digitalis preparation

B, C Feedback: The ACE inhibitors and ARBs reduce the effects of angiotensin II on renal blood flow. They also reduce intraglomerular pressure and may have a renal protective effect in persons with hypertension or type 2 diabetes. However, when combined with diuretics, they may cause prerenal injury in persons with decreased blood flow due to large-vessel or small-vessel kidney disease. Calcium channel blockers are vasodilators.

A 43-year-old female has recently been diagnosed with systemic lupus erythematosus (SLE) glomerulonephritis. She has presented to the out-client department to have a renal biopsy. Knowing the usual treatment options, the nurse should anticipate educating the client (who has a positive biopsy result) on which of the following medications being prescribed? Select all that apply. A) Lasix, a diuretic B) Prednisone, a corticosteroid C) Captopril, an ACE inhibitor D) Ampicillin, an antibiotic

B, C Feedback: Treatment depends on the extent of glomerular involvement. Oral corticosteroids and angiotensin-converting enzyme (ACE) inhibitors are the mainstays of treatment. Diuretics and antibiotics are not part of the treatment protocol.

The physician suspects that a client with kidney stones has developed magnesium ammonium phosphate (struvite) stones based on which of the following urinalysis results? Select all that apply. A) Elevated uric acid levels B) Alkaline urine pH C) High urine phosphate level D) High bacterial count E) Presence of cystine particles

B, C, D Feedback: Magnesium ammonium phosphate stones, also called struvite stones, form only in alkaline urine and in the presence of bacteria that possess an enzyme called urease, which splits the urea in the urine into ammonia and carbon dioxide. The ammonia that is formed takes up a hydrogen ion to become an ammonium ion, increasing the pH of the urine so that it becomes more alkaline. Because phosphate levels are increased in alkaline urine and because magnesium always is present in the urine, struvite stones form. Uric acid stones develop in conditions of gout and high concentrations of uric acid in the urine. Cystine stones account for less than 1% of kidney stones overall but represent a significant proportion of childhood calculi. They are seen in cystinuria, which results from a genetic defect in renal transport of cystine.

While taking a client history, which of the following assessments lead the nurse to suspect the client may have polycystic kidney disease? Select all that apply. A) Massive proteinuria on dipstick urine specimen B) renal colic with flank pain C) bright red blood in urine sample D) elevated blood pressure of 180/94 E) Shortness of breath (SOB) with loud rhonchi and wheezes heard on auscultation

B, C, D Feedback: The manifestations of ADPKD include pain from the enlarging cysts that may reach debilitating levels, episodes of gross hematuria from bleeding into a cyst, infected cysts from ascending UTIs, and hypertension resulting from compression of intrarenal blood vessels with activation of the renin- angiotensin mechanism. Renal colic caused by nephrolithiasis, or kidney stones, occurs in about 20% of persons with ADPKD. One type of pain associated with kidney stones is renal colic, described as colicky pain that accompanies stretching of the collecting system or ureter. Nephrotic syndrome is characterized by massive proteinuria. SOB with abnormal respiratory sounds is not usually associated with ADPKD.

A client has arrived in the emergency department in cardiogenic shock. Which of the following assessment findings confirm this diagnosis? Select all that apply. A) Bright red color noted in the nail beds and lips B) Less than 5 mL dark, concentrated urine in the past hour C) BP reading of 80/65 D) Difficult to arouse with changes in level of consciousness D) Diminished breath sounds in the bases, bilaterally

B, C, D Feedback: The signs and symptoms of cardiogenic shock are consistent with those of end-stage heart failure. The lips, nail beds, and skin may become cyanotic because of stagnation of blood flow. Mean arterial and systolic blood pressures decrease due to poor stroke volume, and there is a narrow pulse pressure because of arterial vasoconstriction. Urine output decreases because of lower renal perfusion pressures and the increased release of aldosterone. Neurologic changes, such as alterations in cognition or consciousness, may occur because of low cardiac output and poor cerebral perfusion.

A family member comes rushing out of a client's room telling the nurse that the loved one can't breathe. The nurse has just left the room after hanging IV penicillin. Which of the following clinical manifestations lead the nurse to suspect the client is experiencing anaphylactic shock? Select all that apply. A) Incontinent of urine B) Severe bronchospasm C) Wheezing sound on inspiration D) Hives over entire body E) Swelling around the lips and eyes

B, C, D, E Feedback: Anaphylactic shock results from an immunologically mediated reaction in which vasodilator substances such as histamine are released into the blood. These substances cause vasodilation of arterioles and venules along with a marked increase in capillary permeability. The vascular response in anaphylaxis is often accompanied by life- threatening laryngeal edema and bronchospasm, circulatory collapse, contraction of gastrointestinal and uterine smooth muscle, and urticaria (hives) or angioedema. The onset and severity of anaphylaxis depend on the sensitivity of the person and the rate and quantity of antigen exposure. Signs and symptoms associated with impending anaphylactic shock include abdominal cramps; apprehension; warm or burning sensation of the skin, itching, and urticaria (i.e., hives); and respiratory distress such as coughing, choking, wheezing, chest tightness, and difficulty in breathing.

The body compensates for obstructed urine outflow up to a certain point. Which of the following signs/symptoms lead the nurse to suspect decompensatory changes are occurring? Select all that apply. A) Reports of renal colic B) Urinary frequency noted C) High residual volume up to 1000 mL D) Postvoid residual volume less than 50 mL E) Must strain to initiate the stream of urine

B, C, E feedback: Compensatory changes to chronic obstruction include increased urge to urinate (urinary frequency). When compensatory mechanism no longer is effective, signs of decompensation begin to appear. The period of detrusor muscle contraction becomes too short to expel the urine completely, and residual urine remains in the bladder. A PVR value of less than 50 mL is considered adequate bladder emptying, and more than 200 mL indicates inadequate bladder emptying. At this point, the symptoms of obstruction—frequency of urination (during day and night), hesitancy, need to strain to initiate urination, a weak and small stream, and termination of the stream before the bladder is completely emptied—become pronounced. Bladder spasms are a symptom of outlet obstruction and do not increase urine output/decrease obstruction.

A chronic kidney disease (CKD) client asks the nurse, "Why do I itch all the time?" The nurse bases her response on which of the following integumentary physiologic factors that causes pruritis? Select all that apply. A) Too harsh of soap while bathing B) Decrease in perspiration C) Limited sodium intake D) Enlarged size of sweat glands E) Elevated serum phosphate levels

B, E Feedback: Dry, itchy skin is a common consequence of CKD. Pruritus is common; it results from the high serum phosphate levels and the development of phosphate crystals that occur with hyperparathyroidism. Harsh soap (may dry the skin), limited Na+ intake, and enlarged sweat glands are not noted to accompany or result in pruritus.

22-16. A 25-year-old cystic fibrosis client presents to the clinic in obvious respiratory distress. Following physical exam, the health care provider suspects bronchiectasis based on which of the following findings? Select all that apply. A) Crushing, substernal chest pain B) Copious amounts of foul-smelling purulent sputum C) Neck vein distention D) Blood-tinged sputum E) Wheezing throughout the lung fields

B,D,E feedback: Bronchiectasis is usually manifested by a chronic productive cough, often with several 100 mL of foul-smelling, purulent sputum a day. Hemoptysis is common. Dyspnea and wheezing occur in about 75% of clients. Crushing substernal chest pain and next vein distention are more suggestive of pulmonary emboli or myocardial infarction with right- sided heart failure.

Clients who have been bed-ridden for a long time likely will experience: A) An inability to produce sufficient amounts of surfactant and may require recombinant forms B) Shallow, quiet breathing, which impairs the spreading of surfactant C) A sharp increase in surfactant levels that will require frequent suctioning D) Increase in their depth of breathing, which increases lung volumes causing more surfactant to spread out over the alveolar surfaces

B. feedback At low lung volumes, the molecules of surfactant become tightly packed, and at higher lung volumes, they spread out to cover the alveolar surface. In surgical clients and bed-ridden persons, shallow and quiet breathing often impairs the spreading of surfactant. Premature infants may require recombinant forms of surfactant to treat infant respiratory distress syndrome. Suctioning cannot be done at the alveolar level. One of the treatments for bedrest clients is cough and deep breathing exercises to enhance the spread of surfactant to prevent atelectasis.

Respiratory movement of air that does not participate in alveolar gas exchange is known as alveolar dead space. Dead space increases when alveolar/alveoli: A) Carbon dioxide level is high. B) Air supply exceeds blood flow. C) Contain pulmonary edema fluid. D) Collapse onto the capillary bed.

B. Air supply exceeds blood flow. Feedback: Alveolar dead space increases when ventilation exceeds perfusion, such as with an embolus obstructing blood flow to the lung area. The oxygen/CO2 gas content in the alveoli has no effect on increased dead space caused by hypoperfusion. Pulmonary edema and alveoli collapse adversely affect gas exchange but have no effect on increased dead space caused by a lack of blood flow

Above the glottis that opens and closes for speech, the epiglottis performs which physiologic functions during swallowing? A) Open the epiglottis B) Cover the larynx C) Collapse the vocal cords D) Constrict the airways

B. Cover the larynx Feedback: During swallowing, the free edges of the epiglottis move downward to cover the larynx, thereby preventing liquids and foods from entering. When substances other than air manage to enter the airway, the vocal folds serve as a sphincter, causing the larynx muscles to constrict and close and/or collapse the airway as a protective measure.

In men experiencing nonrelaxing external sphincter with associated urine retention, the health care worker should assess for which of the following possible causes? A) Increased intra-abdominal pressure B) Chronic prostatitis C) Chronic stress response D) Pelvic inflammatory disease

BFeedback: In men, chronic prostatitis contributes to impaired (nonrelaxing) external sphincter with urine retention. The stress response can cause retention of urine as part of the "fight- or-flight" response, unrelated to sphincter dysfunction. Developmental delays are associated with female or male children (not men). If intra-abdominal pressure increases as it does during actions such as coughing, laughing, or sneezing and if this pressure is not equally transmitted to the urethra, then incontinence occurs. PID is primarily a female disorder.

As a result of hypoxemia and polycythemia, persons with chronic obstructive bronchitis are prone to: A) Breakdown of elastin B) Left-sided heart failure C) Pulmonary hypertension D) Expiratory airway collapse

C Feedback: Hypoxemia causes reflex vasoconstriction of the pulmonary vessels and further impairment of gas exchange in the lung. Hypoxemia also stimulates red blood cell production, causing polycythemia. As a result, persons with chronic obstructive bronchitis may develop pulmonary hypertension and right-sided heart failure. With breakdown and loss of lung elasticity and hyperinflation of the lungs with emphysema, the airways often collapse during expiration.

In the intensive care unit (ICU), the nurse is caring for a trauma client who has abdominal injuries is beginning to have a decrease in BP and increased pulse rate and is pale with diaphoretic skin. The nurse is assessing the client for hemorrhagic shock. If the client is in shock, the nurse would expect to find: A) Excess output of blood-tinged urine B) Complaints of flank pain rotating around the abdominal muscles C) Significant decrease in urine output due to decrease in renal blood flow D) An increase in GFR due to relaxation of the afferent arterioles

C During periods of strong sympathetic stimulation, such as shock, constriction of the afferent arteriole causes a marked decrease in renal blood flow and thus glomerular filtration pressure. Consequently, urine output can fall almost to 0. Unless the injury is specific to the kidney, the client will not have blood in urine and urine production will not be excessive. Flank pain is associated with obstruction due to stone formation. The GFR will decrease rather than increase.

A client who developed a deep vein thrombosis during a prolonged period of bed rest has deteriorated as the clot has dislodged and resulted in a pulmonary embolism. Which of the following types of shock is this client at risk of experiencing? A) Cardiogenic shock B) Hypovolemic shock C) Obstructive shock D) Distributive shock

C Feedback Obstructive shock results from mechanical obstruction of the flow of blood through the central circulation, such as the blockage that characterizes a pulmonary embolism.

An adult client has been diagnosed with polycystic kidney disease. Which of the client's following statements demonstrates an accurate understanding of this diagnosis? A) "I suppose I really should have paid more attention to my blood pressure." B) "I've always been prone to getting UTIs, and now I know why." C) "I suppose I should be tested to see if my children might inherit this." D) "I had a feeling that I was taking too many medications, and now I know the damage they can do."

C Feedback: Autosomal dominant polycystic kidney disease is the most common of all inherited kidney diseases. The disorder is characterized by multiple expanding cysts of both kidneys that ultimately destroy the surrounding kidney structures and cause renal failure. The etiology of polycystic kidney disease (PKD) is not infective, and it is not caused by nephrotoxic drugs or uncontrolled hypertension.

A client with a diagnosis of heart failure has returned from a visit with his primary care provider with a prescription for a change in his daily medication regimen. Which of the following drugs is likely to improve the client's cardiac function by increasing the force and strength of ventricular contractions? A) A β-adrenergic blocker B) A diuretic C) A cardiac glycoside D) An ACE inhibitor

C Feedback: Cardiac glycosides improve cardiac function by increasing the force and strength of ventricular contractions. β-Adrenergic blockers decrease left ventricular dysfunction associated with activation of the sympathetic nervous system. ACE inhibitors block the conversion of angiotensin I to II, whereas diuretics promote the excretion of fluid.

Regardless of the cause, chronic kidney disease results in progressive permanent loss of nephrons and glomerular filtration, and renal: A) Tubule dysplasia B) Vascular pressure C) Endocrine functions D) Hypophosphatemia

C Feedback: Chronic kidney disease results in loss of nephrons, tubule, and endocrine functions such as erythropoietin production. Systemic and renal hypertension is commonly an early manifestation of chronic kidney disease, caused by resistance to blood flow through the constricted renal vessels. Tubule hypertrophy is a compensatory response for those destroyed—when the few remaining nephrons are destroyed, renal failure is apparent. Phosphate accumulates in the blood; since it is inversely related to calcium, the levels of which remain chronically low.

A client who has had recurrent UTIs asks the nurse about the old wise tale of drinking cranberry juice daily. The nurse can respond: A) "There is no research on this topic, so I don't think it will help you." B) "Studies on this are based on a person drinking at least 1 gallon of juice/day." C) "Research suggests cranberry juice will reduce bacterial adherence to the lining of the urinary tract." D) "Beer is probably more effective at killing bacteria than cranberry juice."

C Feedback: Cranberry juice or blueberry juice has been suggested as a preventive measure for persons with frequent UTIs. Studies suggest that these juices reduce bacterial adherence to the epithelial lining of the urinary tract.

The nurse is providing care for a client who has a diagnosis of kidney failure. Which of the following laboratory findings is consistent with this client's diagnosis? A) Elevation in vitamin D levels B) Hypophosphatemia C) Hypocalcemia D) Hypokalemia

C Feedback: Diagnostic findings that are congruent with a diagnosis of kidney failure include hyperphosphatemia, hypocalcemia, a decrease in active vitamin D levels, and secondary hyperparathyroidism.

A client's most recent blood work reveals a blood urea nitrogen (BUN) level of 36 mg/dL (normal range 8 to 25 mg/dL). Which of the following factors may have contributed to this finding? A) Increased salt intake B) Action of ADH C) Dehydration D) Parasympathetic nervous system stimulation

C Feedback: During periods of dehydration, the blood volume and glomerular filtration rate (GFR) drop, and BUN levels increase. Increased salt intake, parasympathetic stimulation, and the action of ADH do not normally result in an increase in BUN.

In anatomy class, the instructor asks, "Explain how urine is expelled from the bladder during voiding." The student with the most accurate response would be: A) "The urothelium acts as a barrier to prevent urine from seeping into capillaries." B) "The beginning of micturition occurs when neurons send messages down to the pudendal nerve." C) "The detrusor muscle contract down on the urine and the ureteral orifices are forced shut. The external sphincter relaxes as urine moves out of the bladder." D) "It's really the external sphincter muscle that controls urination. The somatic nervous system innervates the muscles of the external sphincter and the pelvic floor muscles that together control the outflow of urine."

C Feedback: During the act of micturition, the detrusor muscle of the bladder fundus and bladder neck contracts down on the urine and the ureteral orifices are forced shut. The bladder neck is widened and shortened, and the external sphincter relaxes as urine moves out of the bladder. Descent of the diaphragm and contraction of the abdominal muscles raise intra-abdominal pressure and aid in the expulsion of urine from the bladder.

A client had excessive blood loss and prolonged hypotension during surgery. His postoperative urine output is sharply decreased, and his blood urea nitrogen (BUN) is elevated. The most likely cause for the change is acute: A) Prerenal inflammation B) Bladder outlet obstruction C) Tubular necrosis D) Intrarenal nephrotoxicity

C Feedback: Ischemic acute tubular necrosis (ATN) occurs most frequently in persons who have major surgery with prolonged renal hypoperfusion —this directly damages the tubular epithelial cells with acute suppression of renal function. Nephrotoxic ATN is caused by toxic agents or drugs. Prerenal vasoconstriction is associated with acute-onset loss of renal output. Bladder (postrenal) obstruction would not affect the BUN, since it rarely causes renal failure.

A pregnant woman who is beginning her third trimester has been diagnosed with a urinary tract infection (UTI). Which of the following factors most likely predisposed this client to the development of a UTI? A) Increased urine alkalinity during pregnancy B) Hypertrophy of the bladder wall C) Dilation of the upper urinary structures D) Spastic peristalsis of the ureters

C Feedback: Normal changes in the functioning of the urinary tract that occur during pregnancy predispose to UTIs. These changes involve the collecting system of the kidneys and include dilation of the renal calyces, pelves, and ureters that begins during the first trimester and becomes most pronounced during the third trimester. Bladder hypertrophy, spastic peristalsis, and increased urine pH are not phenomena that are common accompaniments to pregnancy.

A client has been experiencing increasing fatigue in recent months, a trend that has prompted an echocardiogram. Results of this diagnostic test suggest that the client's end- diastolic volume is insufficient. Which of the following parameters of cardiac performance will directly decrease as a result of this? A) Inotropy B) Cardiac contractility C) Preload D) Afterload

C Feedback: Preload is the volume of blood stretching the heart muscle at the end of diastole and is normally determined mainly by the venous return to the heart. Afterload represents the force that the contracting heart muscle must generate to eject blood from the filled heart. Cardiac contractility, or inotropy, is the contractile performance of the heart.

A female client asks, "Why do I leak urine every time I cough or sneeze?" The health care worker's response is based on which physiologic principle? A) Involuntary bladder continence during filling B) A pressure difference between the urethra and bladder C) When intravesical pressure exceeds maximal urethral closure pressure D) A decrease in bladder distensibility

C Feedback: Stress incontinence represents the involuntary loss of urine that occurs when, in the absence of detrusor muscle action, the intravesical pressure exceeds the maximum urethral closure pressure. Stress incontinence, which is a common problem in women of all ages, occurs as the result of weakness or disruption of pelvic floor muscles, leading to poor support of the vesicourethral sphincters. Except during the act of micturition, intraurethral pressure is normally greater than intravesical pressure. Urge incontinence and overactive bladder are associated with urgency caused by bladder infection or CNS or myogenic mechanisms. Overflow incontinence is an involuntary loss of urine that occurs when intravesical pressure exceeds the maximal urethral pressure because of bladder distention in the absence of detrusor activity

An automobile accident client is brought to the emergency department in hypovolemic shock from internal bleeding. Nurses are closely monitoring urine output since a significant decrease signifies that: A) The kidneys are probably injured. B) Renal arteries are clogged with blood. C) The SNS has caused afferent arteries to constrict to decrease blood flow. D) Vagus nerve has caused bradycardia, which decreases amount of blood reaching kidneys.

C Feedback: The afferent and the efferent arterioles are innervated by the sympathetic nervous system and are sensitive to vasoactive hormones, such as angiotensin II. During periods of strong sympathetic stimulation, such as shock, constriction of the afferent arteriole causes a marked decrease in renal blood flow and thus glomerular filtration pressure. Consequently, urine output can fall almost to 0. There is no enough information to conclude the kidneys are injured, the renal arteries are clotted, or the vagus nerve has been innervated.

A client has experienced a bout of coughing after aspirating some of his secretions. The client's coughing was triggered by which of the following? A) Sudden ventilation-perfusion mismatch B) Sudden rise in PCO2 C) Signals from receptors in the tracheobronchial wall D) Signals from central chemoreceptors

C Feedback: The cough reflex is initiated by receptors located in the tracheobronchial wall; these receptors are extremely sensitive to irritating substances and to the presence of excess secretions. Blood gas changes and ventilation-perfusion mismatch do not directly initiate the cough reflex.

Which of the following clients is benefiting from the renin-angiotensin-aldosterone mechanism? A) A teenager who received an injury to the flank area during football practice B) A toddler who was recently diagnosed with cystic fibrosis C) A college student admitted to the neurotrauma unit following traumatic brain injury requiring surgery to evacuate a large hematoma D) A middle-aged adult with osteoarthritis requiring arthroscopic knee surgery to repair a torn meniscus

C Feedback: The kidney releases renin, which enters the blood stream to convert angiotensinogen to angiotensin I. The angiotensin I travels to the lungs, where it is converted to angiotensin II. Angiotensin II acts directly on the kidneys. Renin functions by means of angiotensin II to produce intrarenal vasoconstriction. This helps to regulate blood pressure, which could be a problem for the client having bleeding (hematoma) inside the brain.

When explaining to a CKD client how urea is absorbed, which of the following transport mechanisms will be mentioned? A) Primary active transport B) Secondary active transport C) Passive transport D) Active sodium transport

C Feedback: The mechanisms of transport across the tubular cell membrane are similar to those of other cell membranes in the body and include active and passive transport mechanisms. Water and urea (a by-product of protein metabolism) are passively absorbed along concentration gradients. Sodium (Na+), other electrolytes, as well as urate, glucose, and amino acids, are reabsorbed using primary or secondary active transport mechanisms to move across the tubular membrane. The bulk of energy used by the kidney is for active sodium transport mechanisms that facilitate sodium reabsorption and cotransport of other electrolytes and substances such as glucose and amino acids.

An elderly female client has been hospitalized for the treatment of acute pyelonephritis. Which of the following characteristics of the client is most likely implicated in the etiology of her current health problem? The client: A) Has been diagnosed with type 2 diabetes several years earlier B) Takes a diuretic and an ACE inhibitor each day for the treatment of hypertension C) Recently had a urinary tract infection D) Has peripheral vascular disease

C Feedback: There are two routes by which bacteria can gain access to the kidney: ascending infection from the lower urinary tract and through the bloodstream. Ascending infection from the lower urinary tract is the most important and common route by which bacteria reach the kidney, resulting in acute pyelonephritis. Diabetes, hypertension controlled by a diuretic and an ACE inhibitor, and peripheral vascular disease are not associated with acute pyelonephritis.

An obese, male client with a history of gout and a sedentary lifestyle has been advised by his primary care provider to avoid organ meats, certain fish, and other foods that are high in purines. The care provider is demonstrating an awareness of the client's susceptibility to which of the following types of kidney stones? A) Calcium stones B) Magnesium ammonium phosphate stones C) Uric acid stones D) Cystine stones

C Feedback: Uric acid stones develop in conditions of gout and when high concentrations of uric acid in the urine. Unlike radiopaque calcium stones, uric acid stones are not visible on x-ray films. According to Table 25-2, these stones develop in clients who eat a high-purine diet like Atkins.

The most recent blood work of a client with a diagnosis of heart failure indicates increased levels of atrial natriuretic peptide (ANP) and brain natriuretic peptide (BNP). What is the most likely effect of these peptides on the client's physiology? A) Water retention B) Increased tubular sodium reabsorption C) Inhibition of the renin-angiotensin- aldosterone system D) Sympathetic nervous stimulation

C Feedback:The NPs inhibit the sympathetic nervous system and the renin-angiotensin-aldosterone system, in addition to decreasing tubular sodium and water reabsorption.

The nurse should anticipate that a client who collapsed while running his or her first marathon and has a urine specific gravity of 1.035 is experiencing: A) Frostbite B) Sun stroke C) Dehydration D) Exhaustion

C feedback The usual range of specific gravity is 1.010 to 1.025 with normal fluid intake. Healthy kidneys can produce concentrated urine with specific gravity of 1.030 to 1.040 during periods of dehydration and dilute urine with a specific gravity that approaches 1.000 during periods of too much fluid intake. Frostbite, sunstroke, and exhaustion do not change urine specific gravity if fluid volume is normal.

Which of the following clients is likely at the greatest risk of developing a urinary tract infection? A)A pregnant woman who has been experiencing urinary frequency B) A client with a diagnosis of chronic kidney disease who requires regular hemodialysis C) A 79-year-old client with an indwelling catheter for urinary incontinence D) A confused, 81-year-old client who is incontinent of urine

C feedback There is an increased risk for UTIs in persons with urinary obstruction and reflux, in people with neurogenic disorders that impair bladder emptying, in women who are sexually active, in postmenopausal women, in men with diseases of the prostate, and in elderly persons. Instrumentations and urinary catheterization are the most common predisposing factors for nosocomial, or hospital-acquired UTIs. Frequency and incontinence may be signs and symptoms of UTIs, but they are not causative of the infections.

Glomerulonephritis is usually caused by: A) Vesicoureteral reflux B) Catheter-induced infection C) Antigen-antibody complexes D) Glomerular membrane viruses

C feedback Two types of immune mechanisms have been implicated in the development of glomerular disease: injury resulting from antibodies reacting with fixed glomerular antigens and injury resulting from circulating antigen- antibody complexes that become trapped in the glomerular membrane. Reflux, which is the most common cause of chronic pyelonephritis, results from superimposition of infection on congenital vesicoureteral reflux or intrarenal reflux. Urinary catheters provide a means for microorganisms to ascend into the urinary tract to cause bladder infections or pyelonephritis.

A child is recovering from a bout with group A β-hemolytic Streptococcus infection. They return to the clinic a week later complaining of decrease in urine output with puffiness and edema noted in the face and hands. The health care provider suspects the child has developed: A) Autosomal recessive polycystic kidney disease B) Adult-onset medullary cystic disease C) Acute postinfectious glomerulonephritis D) Acute nephritic syndrome

C Feedback: The classic case of poststreptococcal glomerulonephritis follows a streptococcal infection by approximately 7 to 12 days—the time needed for the development of antibodies. Oliguria, which develops as the GFR decreases, is one of the first symptoms. Proteinuria and hematuria follow because of increased glomerular capillary wall permeability. Sodium and water retention gives rise to edema (particularly of the face and hands) and hypertension. Adults with medullary cystic kidney disease present first with polyuria, polydipsia, and enuresis (bed- wetting), which reflect impaired ability of the kidneys to concentrate urine. The typical infant with ARPKD presents with bilateral flank masses, accompanied by severe renal failure, signs of impaired lung development, and variable degrees of liver fibrosis and portal hypertension. Acute nephritic syndrome is characterized by sudden onset of hematuria, variable degrees of proteinuria, diminished glomerular filtration rate (GFR), oliguria, and signs of impaired renal function.

The primary care provider for a newly admitted hospital client has added the glomerular filtration rate (GFR) to the blood work scheduled for this morning. The client's GFR results return as 50 mL/minute/1.73 m2. The nurse explains to the client that this result represents: A) A need to increase water intake B) The kidneys are functioning normally C) A loss of over half the client's normal kidney function D) concentrated urine

C feedback In clinical practice, GFR is usually estimated using the serum creatinine concentration. A GFR below 60 mL/minute/1.73 m2 represents a loss of one half or more of the level of normal adult kidney function. The GFR is not diagnostic for concentrated urine or the need to drink more water.

A 52-year-old man who is moderately obese has recently been diagnosed with hypertension by his primary care provider. Which of the patient's following statements indicates a need for further health-promotion teaching? A) "I've starting going to the gym before work three times a week." B) "I'm trying to cut back on the amount of salt that I cook with and add to my food." C) "I'm resolving to eat organic foods from now on and to drink a lot more water." D) "I'm planning to lose 15 pounds before the end of this year."

C) "I'm resolving to eat organic foods from now on and to drink a lot more water."

A patient with persistent, primary hypertension remains apathetic about his high blood pressure, stating, "I don't feel sick, and it doesn't seem to be causing me any problems that I can tell." How could a clinician best respond to this patient's statement? A) "Actually, high blood pressure makes you very susceptible to getting diabetes in the future." B) "That's true, but it's an indicator that you're not taking very good care of yourself." C) "You may not sense any problems, but it really increases your risk of heart disease and stroke." D) "You're right, but it's still worthwhile to monitor it in case you do develop problems."

C) "You may not sense any problems, but it really increases your risk of heart disease and stroke."

11. Which of the following assessment findings of a cardiac patient would be suggestive of cardiac tamponade? A) Increasing PaCO2 and decreasing PaO2 B) Audible crackles on chest auscultation and presence of frothy sputum C) 20 mm Hg Drop in systolic blood pressure during respiration D) Normal ECG combined with complaints of chest pain and shortness of breath

C) 20 mm Hg Drop in systolic blood pressure during respiration

A patient who is in a room at one atmosphere (760 mm Hg) is receiving supplementary oxygen therapy that is being delivered at a concentration of 50%. What is the consequent PO2? A) 38,000 mm Hg B) More data are needed C) 380 mm Hg D) 15.2 mm Hg

C) 380 mm Hg

A client who is in a room at 1 atmosphere (760 mm Hg) is receiving supplemental oxygen therapy that is being delivered at a concentration of 50%. What is the consequent PO2? A) 38,000 mm Hg. B) More data are needed. C) 380 mm Hg. D) 15.2 mm Hg.

C) 380 mm Hg. Feedback: The law of partial pressures states that the total pressure of a mixture of gases, as in the atmosphere, is equal to the sum of the partial pressures of the different gases in the mixture. If the concentration of oxygen at 760 mm Hg (1 atmosphere) is 50%, its partial pressure is 380 mm Hg (760 × 0.50).

According to Poiseuille law, airway resistance would be largest in which of the following? A) The trachea B) A bronchus C) A bronchiole D) An alveolus

C) A bronchiole

Which of the following patients should most likely be assessed for orthostatic hypotension? A) A 78-year-old woman who has begun complaining of frequent headaches unrelieved by over-the-counter analgesics. B) A patient whose vision has become much less acute in recent months and who has noticed swelling in her ankles. C) An elderly patient who has experienced two falls since admission while attempting to ambulate to the bathroom. D) A patient who has a history of poorly controlled type 1 diabetes.

C) An elderly patient who has experienced two falls since admission while attempting to ambulate to the bathroom.

A child's history of a recurrent sore throat followed by severe knee and ankle pain has resulted in a diagnostic workup and a diagnosis of rheumatic fever. What are the treatment priorities for this child? A) Cardiac catheterization and corticosteroid therapy B) Implanted pacemaker and β-adrenergic blockers C) Antibiotics and anti-inflammatories D) Pain control and oxygen therapy

C) Antibiotics and anti-inflammatories

A hospital patient was swabbed on admission for antibiotic-resistant organisms and has just been informed that methicillin-resistant Staphylococcus aureus (MRSA) is present in his groin. The patient has a normal core temperature and white blood cell count. This patient is experiencing which of the following? A) Infection B) Proliferation C) Colonization D) Inflammation

C) Colonization

Implantation of a pacemaker is most likely to benefit a patient with which of the following cardiomyopathies? A) Myocarditis B) Takotsubo cardiomyopathy C) Dilated cardiomyopathy (DCM) D) Primary restrictive cardiomyopathy

C) Dilated cardiomyopathy (DCM)

A public health nurse should recognize that sexually transmitted infections (STIs) are typically spread by which of the following mechanisms? A) Penetration B) Vertical transmission C) Direct contact D) Ingestion

C) Direct contact

A 9-month-old infant has been diagnosed with botulism after he was fed honey. The child's mother was prompted to seek care because of this child's sudden onset of neuromuscular deficits, which were later attributed to the release of substances by Clostridium botulinum bacteria. Which virulence factor contributed to this child's illness? A) Endotoxins B) Adhesion factors C) Exotoxins D) Evasive factors

C) Exotoxins

15. Football fans at a college have been shocked to learn of the sudden death of a star player, an event that was attributed in the media to "an enlarged heart." Which of the following disorders was the player's most likely cause of death? A) Takotsubo cardiomyopathy B) Arrhythmogenic right ventricular cardiomyopathy/dysplasia (ARVC/D) C) Hypertrophic cardiomyopathy (HCM) D) Dilated cardiomyopathy (DCM)

C) Hypertrophic cardiomyopathy (HCM)

Because cholesterol is insoluble in plasma, it is mainly carried by the lipoprotein: A) IDL. B) HDL. C) LDL. D) VLDL.

C) LDL.

Which of the following physiologic processes contributes most to the long-term regulation of blood pressure? A) Actions of the renin-angiotensin-aldosterone system B) Release of antidiuretic hormone (vasopressin) by the posterior pituitary C) Renal monitoring and adjustment of extracellular fluid volume D) Integration and modulation of autonomic nervous system (ANS)

C) Renal monitoring and adjustment of extracellular fluid volume

A patient has experienced a bout of coughing after aspirating some of his secretions. The patient's coughing was triggered by which of the following? A) Sudden ventilation-perfusion mismatch B) Sudden rise in PCO2 C) Signals from receptors in the tracheobronchial wall D) Signals from central chemoreceptors

C) Signals from receptors in the tracheobronchial wall

A patient with a long-standing diagnosis of Crohn disease has developed a perianal abscess. Which of the following treatments will this patient most likely require? A) Antiviral therapy B) Antibiotic therapy C) Surgical draining D) Pressure dressing

C) Surgical draining

12. When thinking in terms of airway radius with regard to resistance, the lung structure responsible for the most airway resistance to airflow would be: A) The entire length of trachea B) The right bronchus at the bifurcation C) The bronchioles near the trachea D) A single alveolus

C) The bronchioles near the trachea feedback The primary determinant of airway resistance to airflow is the radius of the conducting airway. Therefore, the site of most of the resistance occurs in the larger bronchioles and bronchi near the trachea, with the smallest airways contributing very little to the total airway resistance. However, although the resistance of each individual bronchiole may be relatively high, their great number results in a large total cross-sectional area, causing their total combined airway resistance to be low.

A patient's exposure to an antibiotic-resistant microorganism while in the hospital has initiated an immune response, a process that is mediated and regulated by cytokines. Which of the following statements is true of cytokines? A) They are stored in the peripheral lymphoid tissues until required. B) They have a long half-life that contributes to an ongoing immune response. C) They are normally released at cell-to-cell interfaces, binding to specific receptors. D) They are capable of performing phagocytosis in the response to viral invasion.

C) They are normally released at cell-to-cell interfaces, binding to specific receptors.

Because of its location, the presence of an abdominal aortic aneurysm may first be noticed as: A) constipation. B) indigestion. C) a pulsating mass. D) mid-abdominal pain.

C) a pulsating mass.

In aortic regurgitation, failure of aortic valve closure during diastole causes an abnormal drop in diastolic pressure. This change in pressure causes decreased: A) stroke volume. B) left ventricular size. C) coronary perfusion. D) arterial pulse pressure.

C) coronary perfusion.

Lung compliance is a measure of the change in ________that occurs with a change in intrapulmonary pressure. A) elastic recoil B) surface tension C) lung volume D) vital capacity

C) lung volume

Congenital heart defects can cause a right heart to left heart shunting of blood that results in increased: A) pulmonary blood volume. B) right ventricle workload. C) unoxygenated blood flow. D) right atrial blood volume.

C) unoxygenated blood flow.

A 40-year-old female with the diagnosis of multiple sclerosis has been experiencing severe bladder spasms along with less bladder volume. This is likely due to: Select all that apply. A) Bladder atonia B) Autonomic hyperreflexia C) Uninhibited bladder D) Neurogenic detrusor overactivity

C, D feedback: Neurogenic detrusor overactivity, or spastic bladder, is usually characterized by reflex bladder spasms and a decrease in bladder volume. The most common causes of neurogenic detrusor overactivity are spinal cord lesions such as spinal cord injury; vascular lesions, tumors, or herniated intervertebral disk; and multiple sclerosis. Bladder atonia is caused by spinal cord injury. Autonomic hyperreflexia is due to spinal cord injuries at the cervical level. Uninhibited bladder can develop after a stroke or during the early stages of multiple sclerosis.

An infant has been diagnosed with autosomal recessive polycystic kidney disease (ARPKD). Which of the following treatment goals would be considered the priority in the care of this child? A) Rehydration therapy B) Total parenteral nutrition C) Prophylactic antibiotics D) Respiratory support

D Feedback: Aggressive ventilatory support is often necessary for neonates with ARPKD, due to the presence of pulmonary hypoplasia and hypoventilation. Hydration, nutrition, and infection prevention are relevant aspects of care, but respiratory interventions are the priority.

Although the distal portion of the urethra often contains pathogens, the urine formed in the kidney and found in the bladder is sterile because of the: A) Alkaline urine B) Glomerular filtering C) Warm temperature D) Washout phenomenon

D Feedback: Although the distal portion of the urethra often contains pathogens, the urine formed in the kidneys and found in the bladder normally is sterile or free of bacteria. This is because of the washout phenomenon, in which urine from the bladder normally washes bacteria out of the urethra during voiding. Because most urinary tract infection (UTI) bacteria ascend through the urethra, glomerular filtration cannot remove it. Pathogens are less likely to survive in acidic conditions; struvite calculi form secondary to bacterial infections causing alkaline urine. The core body temperature is not high enough to destroy bacterial microorganisms in the urine.

A client with a long-standing diagnosis of chronic kidney disease has been experiencing increasing fatigue, lethargy, and activity intolerance in recent weeks. His care team has established that his GFR remains at a low, but stable, level. Which of the following assessments is most likely to inform a differential diagnosis? A) Blood work for white cells and differential B) Cystoscopy and ureteroscopy C) Assessment of pancreatic exocrine and endocrine function D) Blood work for hemoglobin, red blood cells, and hematocrit

D Feedback: Anemia is a frequent, and debilitating, consequence of CKD. The anemia may be due to chronic blood loss, hemolysis, bone marrow suppression due to retained uremic factors, and decrease in red cell production due to impaired production of erythropoietin and iron deficiency. Pancreatic function is not typically affected by CKD, and endoscopic examination is less likely to reveal a cause of fatigue. An infectious etiology is possible and would be informed by white cell assessment, but this is less likely than anemia given the client's complaints.

While explaining the tubular role in reabsorption, the nurse will stress that a diet high in sodium intake will result in sodium reabsorption in which of the following renal locations? A) Descending limb of the loop of Henle B) Ascending limb of the loop of Henle C) Bowman capsule D) Proximal convoluted tubule

D Feedback: Approximately 65% of all reabsorptive and secretory processes that occur in the tubular system take place in the proximal tubule. Electrolytes, such as Na+, K+, Cl-, and bicarbonate (HCO3-), are 65% to 80% reabsorbed in this location.

Impaired skin integrity and skin manifestations are common in persons with chronic kidney disease. Pale skin and subcutaneous bruising are often present as a result of: A) Thrombocytopenia B) Anticoagulant therapy C) Decreased vascular volume D) Impaired platelet function

D Feedback: Bruising and pale skin are present with chronic kidney disease because platelet function is impaired. Adequate platelets are available, but the function is abnormal. Renal clients do not routinely receive anticoagulant therapy, since they already have bleeding tendencies. Increased vascular volume is associated with renal disease.

Diffusion of gases in the lung is decreased, as in pulmonary edema or pneumonia, by causing an increase in alveolar: A) Gas pressure difference B) Size and surface area C) Anatomic shunting of blood D) Capillary membrane thickness

D Feedback: Diffusion is adversely affected by any condition that impedes the movement of gas by thickening the capillary membrane. A decrease in gas pressure difference will slow gas exchange. Increased alveolar size and surface area and shunting will increase diffusion rate.

A client has been given the diagnosis of diffuse glomerulonephritis. They ask the nurse what diffuse means. The nurse responds: A) Only some of the glomeruli are affected B) Only one segment of each glomerulus is involved. C) That the mesangial cells are being affected. D) All glomeruli and all parts of the glomeruli are involved.

D Feedback: Glomerular changes can be diffuse, involving all glomeruli and all parts of the glomeruli; focal, meaning only some of the glomeruli are affected; segmental, involving only a certain segment of each glomerulus; and mesangial, affecting only mesangial cells.

Which of the following individuals is most clearly in need of diagnostic testing for lung cancer? A) A client who has required hospitalization with a fever and the production of copious lung secretions B) A client with a history of secondary tuberculosis who failed to complete his prescribed course of antibiotics C) A woman who complains of recurrent lower respiratory infections and who has sought care for increasing shortness of breath D) A man who demonstrates wasting of the pelvic and shoulder muscles combined with signs of hypercalcemia

D Feedback: Hypercalcemia and wasting of the proximal muscles of the pelvic and shoulder girdles are indicative of the paraneoplastic manifestations of lung cancer. The other cited respiratory conditions warrant follow-up and treatment but are not particularly suggestive of a neoplastic etiology.

While in the ICU, a client's status changes. The health care providers suspect heart failure. Which of the following diagnostic procedures would give the staff information about pulmonary capillary pressures, which will lead to the most appropriate interventions? A) Echocardiography B) Radionuclide ventriculography C) Cardiac magnetic resonance imaging D) Hemodynamic monitoring

D Feedback: Invasive hemodynamic monitoring may be used for assessment in acute, life-threatening episodes of heart failure. These monitoring methods include central venous pressure (CVP), pulmonary artery pressure monitoring, measurements of cardiac output, and intra- arterial measurements of blood pressure. Echocardiography plays a key role in assessing ejection fraction, right and left ventricular wall, wall thickness, ventricular chamber size, valve function, heart defects, and pericardial disease. Radionuclide ventriculography is recommended if there is reason to suspect coronary artery disease or ischemia as the underlying cause for heart failure. Cardiac magnetic resonance imaging and cardiac computed tomography are used to document ejection fraction, ventricular preload, and regional wall motion.

After reviewing the 24-hour intake and output of a hospital client, the nurse suspects that the client may be experiencing flaccid bladder dysfunction. Which of the following diagnostic methods is most likely to confirm or rule out whether the client is retaining urine? A)Blood test for creatinine, blood urea nitrogen, and glomerular filtration rate B) Urine test for culture and sensitivity C) Routine urinalysis D) Measurement of postvoid residual (PVR) by ultrasound

D Feedback: Measurement of PVR can be achieved quickly, accurately, and painlessly by the use of ultrasonography. A PVR value of less than 50 mL is considered adequate bladder emptying, and more than 200 mL indicates inadequate bladder emptying. Urine tests and blood tests will not directly indicate whether a client is experiencing bladder fill with insufficiency in emptying.

A client awaiting a heart transplant is experiencing decompensation of her left ventricle that will not respond to medications. The physicians suggest placing the client on a ventricular assist device (VAD). The client asks what this equipment will do. The health care providers respond: A) "Pull your blood from the right side of the heart and run it through a machine to oxygenate it better, and then return it to your body." B) "Measure the pressures inside your heart continuously to asses pumping ability of your left ventricle." C) "Have a probe at the end of a catheter to obtain thermodilution measures, so cardiac output can be calculated." D) "This device will decrease the workload of the myocardium while maintaining cardiac output and systemic arterial pressure."

D Feedback: Refractory heart failure reflects deterioration in cardiac function that is unresponsive to medical or surgical interventions. Ventricular assist devices (VADs) are mechanical pumps used to support ventricular function. VADs are used to decrease the workload of the myocardium while maintaining cardiac output and systemic arterial pressure. This decreases the workload on the ventricle and allows it to rest and recover. The rest of the distractors relate to the monitoring in an ICU of cardiac functioning. Invasive hemodynamic monitoring may be used for assessment in acute, life-threatening episodes of heart failure. With the balloon inflated, the catheter monitors pulmonary capillary pressures (i.e., pulmonary capillary wedge pressure or pulmonary artery occlusion pressure), which reflect pressures from the left ventricle. The pulmonary capillary pressures provide a means of assessing the pumping ability of the left ventricle. One type of pulmonary artery catheter is equipped with a thermistor probe to obtain thermodilution measurements of cardiac output.

A nurse is performing client health education with a 68-year-old man who has recently been diagnosed with heart failure. Which of the following statements demonstrates an accurate understanding of his new diagnosis? A) "I'll be sure to take my beta blocker whenever I feel short of breath." B) "I'm going to avoid as much physical activity as I can so that I preserve my strength." C) "I know it's healthy to drink a lot of water, and I'm going to make sure I do this from now on." D) "I'm trying to think of ways that I can cut down the amount of salt that I usually eat.

D Feedback: Salt and fluid restrictions are indicated for most clients with heart failure (HF). Beta blockers do not address shortness of breath, and cardiac medications are not normally taken in response to acute symptoms. Clients should be encouraged to maintain, and increase, physical activity within the limits of their condition.

Protein and blood cell leakage into the filtrate that occurs in many forms of glomerular disease is a result of changes in the structure and function of the glomerular: A) Renal corpuscle B) Bowman capsule C) Peritubular network D) Basement membrane

D Feedback: The basement membrane is a meshwork of collagen fibers, with slit pores between the fibers creating size-dependent permeability that (normally) does not allow large molecules, such as protein and blood cells through. The renal corpuscle contains the Bowman capsule and capillaries surrounding it. Peritubular capillary network is a low- pressure reabsorptive system that permits rapid fluid/solute transfer to/from the tubules.

Many factors contribute to the incontinence that is common among the elderly. A major factor is increased: A) Detrusor muscle function B) Intake of liquids and water C) Urethral closing pressure D) Use of multiple medications

D Feedback: Use of multiple medications for other health problems can affect bladder function, especially diuretics. Drugs such as hypnotics, tranquilizers, and sedatives can interfere with the conscious inhibition of voiding, leading to urge incontinence. Detrusor muscle function and urethral closing pressure are decreased in the elderly, causing incontinence. Decreased fluid and water intake causes problems of bowel impaction and urinary tract infection.

A young child has been diagnosed with Wilms tumor after his mother discovered an unusual mass, prompting a diagnostic workup. Which of the following characteristics is typical of Wilms tumor? The tumor is usually: A) Asymptomatic B) Self-limiting C) A secondary neoplasm D) Encapsulated

D Feedback: Wilms tumor usually is a solitary mass that occurs in any part of the kidney. It usually is sharply demarcated and variably encapsulated. It is not a self-limiting health problem, and chemotherapy, radiotherapy, and/or surgery may be utilized. Symptoms include hypertension, abdominal pain, and vomiting.

Severe shock can be followed by acute lung injury/acute respiratory distress syndrome (ALI/ARDS) characterized by: A) Hyperventilation B) Excessive surfactant C) Hyperinflated alveolar sacs D) Ventilation-perfusion mismatch

D feedback; Despite the delivery of high levels of oxygen using high-pressure mechanical ventilatory support and positive end-expiratory pressure, many persons with ALI/ARDS remain hypoxic, often with a fatal outcome. Arterial blood gas analysis establishes the presence of profound hypoxemia with hypercapnia, resulting from impaired matching of ventilation and perfusion and from the greatly reduced diffusion of blood gases across the thickened alveolar membranes. Abnormalities in the production, composition, and function of surfactant may contribute to alveolar collapse and gas exchange abnormalities.

22-10. Which of the following manifestations typically accompanies an asthmatic attack? A) Decreased residual volume B) Decreased pulmonary arterial pressure C) Prolonged inspiration D) Hyperinflation of the lungs

D) Hyperinflation of the lungs feedback During a prolonged attack, air becomes trapped behind the occluded and narrowed airways, causing hyperinflation of the lungs. This produces an increase in the residual volume of the lungs. Pulmonary arterial pressure tends to increase and expiration becomes prolonged

A patient has begun taking acyclovir, an antiviral medication, to control herpes simplex outbreaks. What is this drug's mechanism of action? A) Inhibition of viral adhesion to cells B) Elimination of exotoxin production C) Antagonism of somatic cell binding sites D) Interference with viral replication processes

D) Interference with viral replication processes

A client has just returned from his surgical procedure. During initial vital sign measurements, the nurse notes that the client's heart rate is 111 beats/minute and the BP is 100/78 (borderline low). In this early postoperative period, the nurse should be diligently monitoring the client for the development of: A) Pulmonary embolism due to development of deep vein thrombosis B) Side effects from versed administration causing excessive vasoconstriction C) Renal failure due to an overdose of medication D) Hypovolemic shock due to acute intravascular volume loss

D Hypovolemic shock is characterized by diminished blood volume such that there is inadequate filling of the vascular compartment. Hypovolemic shock also can result from an internal hemorrhage or from third-space losses, when extracellular fluid is shifted from the vascular compartment to the interstitial space or compartment, without fluid movement in/out of the cells. Within seconds after the onset of hemorrhage or the loss of blood volume, compensatory manifestations of tachycardia, vasoconstriction, and other signs of sympathetic and adrenal medullary activity appear. There is no indication that this client has developed a pulmonary embolism, is having side effects from versed administration, or is going into renal failure due to an overdose of medication.

Which of the following individuals likely faces the greatest risk for the development of chronic kidney disease? A) A first-time mother who recently lost 1.5 L of blood during a postpartum hemorrhage B) A client whose diagnosis of thyroid cancer necessitated a thyroidectomy C) A client who experienced a hemorrhagic stroke and now has sensory and motor deficits D) A client with a recent diagnosis of type 2 diabetes who does not monitor his blood sugars or control his diet

D feedback Chronic kidney disease (CKD) is a pathophysiologic process that results in the loss of nephrons and a decline in renal function that has persisted for more than 3 months. CKD can result from diabetes, hypertension, glomerulonephritis, lupus (SLE), and polycystic kidney disease. The prevalence and incidence of CKD continue to grow, reflecting the growing elderly population and the increasing number of people with diabetes and hypertension. Hemorrhage may result in acute renal failure, but it is not associated with chronic kidney disease. Stroke and loss of the thyroid gland are not noted to underlie cases of chronic kidney disease.

Which of the following aspects of kidney function is performed by the juxtaglomerular apparatus? A) Regulating urine concentration B) Facilitating active transport to reabsorb electrolytes C) Regulating sodium and potassium elimination D) Matching changes in GFR with renal blood flow

D feedback The juxtaglomerular apparatus is thought to represent a feedback control system that links changes in the glomerular filtration rate (GFR) with renal blood flow.

Which of the following factors is likely to result in decreased renal blood flow? A) Action of dopamine B) Release of nitric oxide C) Action of prostaglandins D) Sympathetic nervous system stimulation

D feedback: Sympathetic nervous system (SNS) stimulation results in decreased renal blood flow by vasoconstriction. Dopamine, nitric oxide, and prostaglandins are all vasodilators.

Clients with chronic obstructive lung disease (COPD) may experience airway closure at the end of normal instead of low lung volumes, which result in: A) Airway constriction of the main bronchus B) Release of epinephrine, a catecholamine, which causes airway dilation C) An increase in the physiological dead space in alveoli that are perfused but not ventilated D) Trapping of large amounts of air that cannot participate in gas exchange

D . Trapping of large amounts of air that cannot participate in gas exchange Feedback: The pressure needed to overcome the tension in the wall of a sphere or an elastic tube is inversely related to its radius; therefore, the small airways close first, trapping some air in the alveoli. This trapping of air may be increased in older persons and persons with chronic lung disease owing to a loss in the elastic recoil properties of the lungs. In these persons, airway closure occurs at the end of normal instead of low lung volumes, trapping larger amounts of air that cannot participate in gas exchange. It does not result in airway constriction, release of catecholamines, or an increase in the physiological dead space.

Completion of a patient's pulmonary function study has yielded the following data: tidal volume: 500 mL; inspiratory reserve: 3100 mL; expiratory reserve: 1200 mL; residual volume: 1200 mL; functional residual capacity: 2400 mL. What is this patient's inspiratory capacity? A) 5500 mL B) 2600 mL C) More data are needed D) 3600 mL

D) 3600 mL Feedback: Inspiratory capacity is the sum of inspiratory reserve volume and tidal volume.

A postsurgical patient's complaints of calf pain combined with the emergence of swelling and redness in the area have culminated in a diagnosis of deep vein thrombosis. What treatment options will be of greatest benefit to this patient? A) Analgesics and use of a pneumatic compression device B) Massage followed by vascular surgery C) Frequent ambulation and the use of compression stockings D) Anticoagulation therapy and elevation of the leg

D) Anticoagulation therapy and elevation of the leg

Persistent cyanosis has led an infant's care team to suspect a congenital heart defect. Which of the following assessment findings would suggest coarctation of the infant's aorta? A) The child has a split S2 heart sound on auscultation. B) ECG reveals atrial fibrillation. C) The child experiences apneic spells after feeding. D) Blood pressure in the child's legs is lower than in the arms.

D) Blood pressure in the child's legs is lower than in the arms.

A serum marker for systemic inflammation, _______, is now considered a major risk factor marker for atherosclerosis, and vascular disease. A) leukocytosis B) homocysteine C) serum lipoprotein D) C-reactive protein

D) C-reactive protein

A respiratory therapist has asked a patient to breathe in as deeply as possible during a pulmonary function test. Inspiration is normally the result of which of the following phenomena? A) Decreased intrapulmonary pressure B) Increased airway pressure C) Increased intrapleural pressure D) Decreased intrathoracic pressure

D) Decreased intrathoracic pressure

A respiratory therapist has asked a client to breathe in as deeply as possible during a pulmonary function test. Inspiration is normally the result of which of the following phenomena? A) Decreased intrapulmonary pressure B) Increased airway pressure C) Increased intrapleural pressure D) Decreased intrathoracic pressure

D) Decreased intrathoracic pressure Feedback: During inspiration, contraction of the diaphragm and expansion of the chest cavity produce a decrease in intrathoracic pressure, causing air to move into the lungs.

A patient's primary care provider has ordered direct antigen detection in the care of a patient with a serious symptomatology of unknown origin. Which of the following processes will be conducted? A) Detecting DNA sequences that are unique to the suspected pathogen B) Growth of biofilms on various media in the laboratory setting C) Quantification of IgG and IgM antibodies in the patient's blood D) Introduction of monoclonal antibodies to a blood sample from the patient

D) Introduction of monoclonal antibodies to a blood sample from the patient

Which of the following would participate in the innate immune response to an infectious microorganism? A) T lymphocytes B) Antibodies C) B lymphocytes D) Neutrophils

D) Neutrophils

A 33-year-old patient who is a long-term intravenous user of heroin has been recently diagnosed with hepatitis C. Which of the following portals of entry most likely led to the patient's infection? A) Direct contact B) Vertical transmission C) Ingestion D) Penetration

D) Penetration

A patient with a diagnosis of community- acquired pneumonia is producing copious secretions that are physically obstructing her airway. Which of the following pathophysiologic processes will result from this condition? A) Compensatory vasoconstriction B) Ventilation without perfusion C) Dead air space D) Perfusion without ventilation

D) Perfusion without ventilation

20. Which of the following assessment findings of a cyanotic infant is incongruent with a diagnosis of tetralogy of Fallot? A) The child has ventricular septal defect. B) The infant's pulmonary outflow channel is narrowed. C) The child has right ventricular hypertrophy. D) The infant's aorta is narrowed.

D) The infant's aorta is narrowed.

A neighbor is complaining to a friend (who happens to be a nurse) about several changes in their body. Which of the following complaints raises a "red flag" because it could be a sign of epithelial cell bladder cancer? A)"Seems like I'm holding onto more water these days." B) "Every now and then, I have urine leak when I cough." C) "Sometimes I get a sharp pain in my side while exercising." D) "I noticed my urine is pinkish red, but I'm not having any pain when I pee."

D) The most common sign of bladder cancer is intermittent painless hematuria. Fluid retention, stress incontinence, and pain with exercise are not usual signs of cancer.

Major histocompatibility complex (MHC) molecules, with human leukocyte antigens (HLAs), are markers on all nucleated cells and have an important role in: A) identifying blood types. B) cell membrane transport. C) suppressing viral replication. D) avoiding transplant rejections.

D) avoiding transplant rejections.

Diffusion of gases in the lung is decreased, as in pulmonary edema or pneumonia, by causing an increase in alveolar: A) gas pressure difference. B) size and surface area. C) anatomic shunting of blood. D) capillary membrane thickness.

D) capillary membrane thickness.

The patient is immobilized following a hip injury and has begun demonstrating lower leg discoloration with edema, pain, tenderness, and increased warmth in the mid-calf area. He has many of the manifestations of: A) stasis ulcerations. B) arterial insufficiency. C) primary varicose veins. D) deep vein thrombosis.

D) deep vein thrombosis.

Bacteria on a sliver in a boy's finger have initiated an adaptive immune response. The boy's lymphocytes and antibodies recognize immunologically active sites on the bacterial surfaces known as: A) Toll-like receptors. B) opsonins. C) chemokines. D) epitopes.

D) epitopes.

Small-vessel vasculitides, a group of vascular disorders that cause vasculitis, are mainly mediated by: A) infectious agents. B) tissue necrosis. C) mononuclear cells. D) hypersensitivity reactions.

D) hypersensitivity reactions.

The effector function of activated members of the complement system includes all of the following EXCEPT: A) chemotaxis. B) opsonization. C) pathogen lysis. D) phagocytosis.

D) phagocytosis.

Activation of lymphocytes is dependent upon the ________ and ________ of the antigens by macrophages. A) memory; clustering B) capture; destruction C) recognition; grouping D) processing; presentation

D) processing; presentation

Generalized acute hypoxia in lung tissue, when alveolar oxygen levels drop below 60 mm Hg, causes pulmonary: A) vasospasms. B) hypertension. C) emboli formation. D) vasoconstriction.

D) vasoconstriction

Endocarditis and rheumatic heart disease are both cardiac complications of systemic infections. Characteristics include a new or changed heart murmur caused by: A) chronic atrial fibrillation. B) myocardial inflammation. C) left ventricle hypertrophy. D) vegetative valve destruction.

D) vegetative valve destruction.

While discussing carbon dioxide transport within the body, the instructor asks, "What enzyme helps carbon dioxide with water to form bicarbonate?" Which student response is correct? A) Dihydrofolate reductase B) Orotidine 5'-phosphate decarboxylase C) Neuraminidase D) Carbonic anhydrase

D. Carbonic anhydrase Feedback: Most of the carbon dioxide diffuses into the red blood cells, where it either forms carbonic acid or combines with hemoglobin. Carbonic acid (H2CO3) is formed when carbon dioxide combines with water (CO2 + H2O = H+ + HCO3-). The process is catalyzed by an enzyme called carbonic anhydrase, which is present in large quantities in red blood cells. Carbonic anhydrase increases the rate of the reaction between carbon dioxide and water approximately 5000-fold. Carbonic acid readily ionizes to form bicarbonate (HCO3-) and hydrogen (H+) ions.

The sensation of chest tightness due to an impending asthmatic attack appears to be related to which of the following physiological causes? A) Transmission of excessive chemoreceptor stimulation of the medullary respiratory center. B) Uncomfortable work of breathing sensation is thought to be mediated by excessive input from stretch receptors in the chest muscles. C) A perceived shortness of breath is a multidimensional sensation involving both the sensory intensity and unpleasantness. D) Input from lung receptors that monitor bronchial constriction.

D. Input from lung receptors that monitor bronchial constriction. feedback The sensation of chest tightness due to an impending asthmatic attack appears to be related to input from lung receptors that monitor bronchial constriction. Transmission of excessive chemoreceptor stimulation of the medullary respiratory center to sensory centers in the forebrain relates to air hunger. Labored breathing, an uncomfortable work of breathing sensation, is thought to be mediated by excessive input from stretch receptors in the chest muscles. A perceived shortness of breath, known as dyspnea, is a multidimensional sensation involving both the sensory intensity and unpleasantness.

Following surgery, a client had a chest x-ray that reported some opacities in the lung bases likely due to atelectasis. Which of the following pathophysiologic processes will result from this condition? A) Compensatory vasoconstriction B) Ventilation without perfusion C) Dead air space D) Perfusion without ventilation

D. Perfusion without ventilation Feedback: With shunt, there is perfusion without ventilation, resulting in a low ventilation- perfusion ratio. This occurs in conditions such as atelectasis in which there is airway obstruction. Ventilation without perfusion (dead air space) is a consequence of impaired pulmonary circulation. Hypoxemia will result in vasodilation, not vasoconstriction.

Generalized acute hypoxia in lung tissue, when alveolar oxygen levels drop below 60 mm Hg, causes pulmonary: A) Vasospasms B) Hypertension C) Embolus formation D) Vasoconstriction

D. Vasoconstriction Feedback: Vasoconstriction of pulmonary vessels is a compensatory response to generalized lung tissue hypoxia. Embolus formation causes a localized hypoxic response. Vasospasms are not involved with the hypoxia response by lung tissue. Pulmonary hypertension is the result of chronic lung tissue hypoxia.

5. On the second or third day after an acute myocardial infarction, the area of necrosis is: a) soft and yellow. b) acutely inflamed. c) granulation tissue. d) fibrous scar tissue.

b) acutely inflamed.

Atherosclerotic plaque is most likely to be unstable and vulnerable to rupture when the plaque has a thin fibrous cap over a: a) red thrombus. b) large lipid core. c) calcified lesion. d) vessel wall injury.

b) large lipid core.

8. Dilated cardiomyopathy with left ventricular dysfunction is characterized by increased wall: a) rigidity. b) thinning. c) thickness. d) contractility.

b) thinning.

In adults, sudden death from an acute myocardial infarction is usually caused by: a) acute myocarditis. b) high troponin levels. c) acute ventricular arrhythmia. d) hypertrophic cardiomyopathy.

c) acute ventricular arrhythmia.

Cardiac tamponade and pericardial effusion can be life-threatening when the pericardial sac _______ and ______ the heart. a) ruptures; releases b) thickens; stretches c) contracts; friction rubs d) fills rapidly; compresses

d) fills rapidly; compresses


Ensembles d'études connexes

Mkrt 301 chap 3 Test and Quiz Questions

View Set

Word Religions- Indigenous Religions

View Set

Essential Cell Biology, Chapter 11, Membrane Stricture

View Set

Chapter 16: Therapy and Treatment

View Set

Bone growth, remodeling and repair

View Set

6th Grade Chapter 5 Test Practice

View Set

PSY 242- Abnormal Psychology Final Exam

View Set

English Alphabet for Kindergarten

View Set

Chapter 9 Online Retail & Services Matching

View Set